You are on page 1of 58

Total Marks : 200

Online Prelims TEST - 3 (SUBJECT WISE)


( InsightsIAS Mock Test Series for UPSC Preliminary Exam 2020 )

1 With reference East India Company, the Anglicist-Orientalist controversy was known in the context of

A. Railways
B. Education
C. Trade
D. Foreign Policy

Your Answer :
Correct Answer : B

Answer Justification :

ORIENTALIST-ANGLICIST CONTROVERSY:

Within the General Committee on Public Instruction, the Anglicists argued that the government
spending on education should be exclusively for modern studies.

The Orientalists said while western sciences and literature should be taught to prepare students to
take up jobs, emphasis should be placed on expansion of traditional Indian learning.

Even the Anglicists were divided over the question of medium of instruction—one faction was for
English language as the medium, while the other faction was for Indian languages (vernaculars) for
the purpose.

Unfortunately there was a great deal of confusion over English and vernacular languages as media
of instruction and as objects of study.

Lord Macaulay's Minute (1835) settled the row in favour of Anglicists—the limited government
resources were to be devoted to teaching of western sciences and literature through the medium of
English language alone.

Hence, OPTION (b) is correct.

2 The Ahom Revolt against the British was led by

A. Rao Bharamal
B. Gomdhar Konwar
C. Tomma Dorasanyasi
D. Birsa Munda

Your Answer : B
Correct Answer : B

Answer Justification :

The British had pledged to withdraw after the First Burma War (1824-26) from Assam. But, after
the war, instead of withdrawing, the British attempted to incorporate the Ahoms' territories in the
Company's dominion. This sparked off a rebellion in 1828 under the leadership of Gomdhar

www.insightsactivelearn.com 1
Total Marks : 200
Online Prelims TEST - 3 (SUBJECT WISE)
( InsightsIAS Mock Test Series for UPSC Preliminary Exam 2020 )

Konwar. Finally, the Company decided to follow a conciliatory policy and handed over Upper
Assam to Maharaja Purandar Singh Narendra and part of the kingdom was restored to the
Assamese king.

Hence option B is correct.

3 Consider the following statements:


1. Saralabala Devi Chaudharani is associated with Bharat Stree Mandal.
2. Mirza Ghulam Ahmed founded the Ahmadiya Movement.

Which of the statements given above is/are correct?


A. 1 only
B. 2 only
C. Both 1 and 2
D. Neither 1 nor 2

Your Answer : C
Correct Answer : C

Answer Justification :

Both the statements are correct.

Bharat Stree Mahamandal was the first women's organization in India founded by Sarala Devi
Chaudhurani in Allahabad in 1910. One of the primary goals of the organization was to
promote female education. Hence, Statement 1 is correct.

Ahmadiya Movement was founded by Mirza Ghulam Ahmed in 1889. It was based on liberal
principles. It described itself as the standard-bearer of Mohammedan Renaissance, and based itself,
like the Brahmo Samaj, on the principles of universal religion of all humanity, opposing jihad
(sacred war
against non-Muslims). Hence, statement 2 is correct.

4 Which of the following series of treaties is arranged in correct chronological order?

A. Treaty of Allahabad –Treaty of Srirangapatnam – Treaty of Bassein


B. Treaty of Srirangapatnam – Treaty of Allahabad – Treaty of Bassein
C. Treaty of Bassein – Treaty of Allahabad – Treaty of Srirangapatnam
D. Treaty of Srirangapatnam – Treaty of Bassein – Treaty of Allahabad

Your Answer :
Correct Answer : A

Answer Justification :

The Treaty of Allahabad was signed on 12 August 1765, between the Mughal Emperor Shah
Alam II, son of the late Emperor Alamgir II, and Robert Clive, of the East India Company, as a

www.insightsactivelearn.com 2
Online Prelims TEST - 3 (SUBJECT WISE)
( InsightsIAS Mock Test Series for UPSC Preliminary Exam 2020 )

result of the Battle of Buxar of 22 October 1764.

The Treaty of Seringapatam was signed on 18 March 1792 at the end of the Third Anglo-
Mysore War. Its signatories included Lord Cornwallis on behalf of the British East India
Company, representatives of the Nizam of Hyderabad and the Maratha Empire, and Tipu Sultan,
the ruler of Mysore

The Treaty of Bassein was a pact signed on 31 December 1802 between the British East India
Company and Baji Rao II, the Maratha Peshwa of Pune in India after the Battle of Poona.

Hence, option (a) is correct.

5 Consider the following statements:


1. Zamindari System was introduced by Thomas Munro in 1793 through Permanent Settlement Act.
2. Ryotwari System was introduced in the provinces of Bengal, Bihar and Orissa.

Which of the statements given above is/are correct?


A. 1 only
B. 2 only
C. Both 1 and 2
D. Neither 1 nor 2

Your Answer : C
Correct Answer : D

Answer Justification :

Both the statements are INCORRECT.

Zamindari System was introduced by Cornwallis in 1793 through Permanent Settlement Act. It
was introduced in provinces of Bengal, Bihar, Orissa and Varanasi.

Zamindars were recognized as owner of the lands. Zamindars were given the rights to collect the
rent from the peasants. The realized amount would be divided into 11 parts. 1/11 of the share
belongs to Zamindars and 10/11 of the share belongs to East India Company

Hence, statement 1 is incorrect.

Ryotwari System was introduced by Thomas Munro in 1820. Major areas of introduction
include Madras, Bombay, parts of Assam and Coorg provinces of British India. In Ryotwari
System the ownership rights were handed over to the peasants. British Government collected taxes
directly from the peasants.

Hence, statement 2 is incorrect.

6 With reference to the provisions of Regulating Act, 1773, consider the following statements:
1. A supreme court of four English judges was set up in Bombay.
2. The appointment of a governor-general of Fort William in Bengal with supervisory powers over the

www.insightsactivelearn.com 3
Online Prelims TEST - 3 (SUBJECT WISE)
( InsightsIAS Mock Test Series for UPSC Preliminary Exam 2020 )

presidencies of Madras and Bombay.

Which of the statements given above is/are correct?


A. 1 only
B. 2 only
C. Both 1 and 2
D. Neither 1 nor 2

Your Answer : C
Correct Answer : B

Answer Justification :

Regulating Act, (1773), legislation passed by the British Parliament for the regulation of the
British East India Company’s Indian territories, mainly in Bengal. It was the first intervention by the
British government in the company’s territorial affairs and marked the beginning of a takeover
process that was completed in 1858.

The main provisions of the act were

The appointment of a governor-general of Fort William in Bengal with supervisory


powers over the presidencies of Madras (now Chennai) and Bombay (now Mumbai).
Hence statement 2 is correct.

The governor-general had a council of four and was given a casting vote but no veto.

A supreme court of four English judges was set up in Calcutta (now Kolkata). Hence
statement 1 is incorrect.

In Great Britain annual elections of 24 directors were replaced by the election of six judges a
year, each for a four-year term, and the qualification for a vote was raised from £500 to
£1,000.

7 Consider the following statements:


1. On 20th May, 1498, Portuguese sailor named Vasco da Gama arrived at Calicut.
2. Alfonso de Albuquerque captured Goa from the Sultan of Golconda.

Which of the statements given above is/are correct?


A. 1 only
B. 2 only
C. Both 1 and 2
D. Neither 1 nor 2

Your Answer : C

www.insightsactivelearn.com 4
Online Prelims TEST - 3 (SUBJECT WISE)
( InsightsIAS Mock Test Series for UPSC Preliminary Exam 2020 )

Correct Answer : A

Answer Justification :

The Portuguese were the first European community to discover a direct sea route to India. On 20th
May, 1498, a Portuguese sailor named Vasco da Gama arrived at Calicut, an important sea
port of South-West India. He was cordially received by King Zamorin, the local ruler, and was
offered certain privileges. Hence, statement 1 is correct.

The real founder of the Portuguese power in India was Alfonso de Albuquerque. In 1509, he
replaced Almeida as the governor and in the next year (1510), he captured Goa from the
Sultan of Bijapur. Goa subsequently became the headquarters of the Portuguese settlements in
India. Hence statement 2 is incorrect.

8 Consider the following statements:


1. Charter Act of 1813 provided for the first time, setting apart a sum of one lakh rupees for the
spread of education in India.
2. Charter Act of 1833 changed designation of the Governor-General of Bengal was changed to
Governor-General of India.

Which of the statements given above is/are correct?


A. 1 only
B. 2 only
C. Both 1 and 2
D. Neither 1 nor 2

Your Answer : C
Correct Answer : C

Answer Justification :

Both the statements are correct.

Charter Act, 1813 terminated the Company’s monopoly of trading in India. The Indian trade was
now open to all British subjects. However, the Company’s trade monopoly in China was retained.
The Act also provided for the first time, setting apart a sum of one lakh rupees for the
spread of education in India. Hence, statement 1 is correct.

Again in 1833 the Charter was brought before the Parliament for renewal. The Act was passed on
August 28, 1833 and put into force on 22nd April 1834.The Act ended the commercial functions of
the Company. However it was allowed to retain its administrative and political powers.

The Act removed all restriction on the immigration of British subjects to India for trade, missionary
work or any other lawful purpose. It empowered the Governor-General to make laws and
regulations for the welfare of its Indian subjects. The Act further strengthened the hands of the
Central Government over the Presidency Governments. The designation of the Governor-
General of Bengal was changed to Governor-General of India. He was invested with full power
and authority to control civil and military administration of the Presidency Governments. Hence

www.insightsactivelearn.com 5
Online Prelims TEST - 3 (SUBJECT WISE)
( InsightsIAS Mock Test Series for UPSC Preliminary Exam 2020 )

statement 2 is correct.

9 Consider the following statements:


1. In 1800 Fort William College was established at Calcutta during the Governor-Generalship of Lord
Cornwallis.
2. Charter Act of 1853 decreed that all recruits to the civil service were to be selected through a
competitive examination.

Which of the statements given above is/are correct?


A. 1 only
B. 2 only
C. Both 1 and 2
D. Neither 1 nor 2

Your Answer :
Correct Answer : B

Answer Justification :

In 1800 Fort William College was established at Calcutta during the Governor-Generalship
of Lord Wellesley to train the young officers recruited for civil services. In 1806, the Directors of
the Company shifted the venue of the training of civil service officers from Calcutta to the East
Indian College at Haileybury in England. Hence statement 1 is incorrect.

Merit was made the basis of selection to the public services for the first time by the Charter Act of
1833. A committee was appointed under Lord Macaulay in 1834 for implementation of this
provision. The committee suggested fixing the maximum age for eligibility to the examination at 18
years. But it was not implemented. Till 1853 all appointments to the civil service were made
by the Directors of the East India Company. But the Charter Act of 1853 decreed that all
recruits to the civil service were to be selected through a competitive examination. Hence,
statement 2 is correct.

10 Consider the following statements:


1. Warren Hastings established a Diwani Adalat and a Faujdari Adalat in every province.
2. Lord Cornwallis abolished the Provincial Court of Appeal and Circuit.

Which of the statements given above is/are correct?


A. 1 only
B. 2 only
C. Both 1 and 2
D. Neither 1 nor 2

Your Answer : A
Correct Answer : D

Answer Justification :

www.insightsactivelearn.com 6
Online Prelims TEST - 3 (SUBJECT WISE)
( InsightsIAS Mock Test Series for UPSC Preliminary Exam 2020 )

The British laid the foundation of a new judicial system in India. Warren Hastings was the first to
establish the modern judicial system in place of the traditional one. He established a Diwani
Adalat and a Faujdari Adalat in every district. The Diwani Adalat was presided over by the
District Collector who belonged to the civil service. The Faujdari Adalats were presided over by
Indian officers who decided the cases with the assistance of Qazis and Muftis.

Hence, statement 1 is incorrect.

In 1831 William Bentinck abolished the Provincial Court of Appeal and Circuit. Their work
was first assigned to Commissions. Later it was divided among the District Judges and District
Collectors. Hence, statement 2 is incorrect.

11 Which of the following reforms/events was/were undertaken by Governor General William Bentick?
1. Abolition of sati
2. Suppression of thugi
3. Vellore Mutiny
4. First Afghan War

Select the correct answer using the code given below:


A. 1 and 2 only
B. 1, 2 and 4 only
C. 1, 3 and 4 only
D. 1, 2, 3 and 4

Your Answer : B
Correct Answer : A

Answer Justification :

Some of the reforms/events that took place under the Governor General William Bentick(1828-1835)
were

(i) Abolition of sati and other cruel rites (1829)

(ii) Suppression of thugi (1830).

(iii) Charter Act of 1833.


(iv) Resolution of 1835, and educational reforms and introduction of English as the official language
etc

Thus statement 1 and statement 2 are correct.

Vellore Mutiny, outbreak against the British on July 10, 1806, by sepoys (Indian troops
employed by the British) at Vellore. The incident began when the sepoys broke into the fort where
the many sons and daughters of Tippu Sultan of Mysore and their families had been lodged since
their surrender at Seringapatam (now Shrirangapattana) in 1799 during the fourth Mysore War. It
took place during the tenure of Sir George Barlow. Hence, statement 3 is incorrect.

www.insightsactivelearn.com 7
Online Prelims TEST - 3 (SUBJECT WISE)
( InsightsIAS Mock Test Series for UPSC Preliminary Exam 2020 )

The First Anglo-Afghan War was fought between the British East India Company and the Emirate of
Afghanistan from 1839 to 1842. Initially, the British successfully intervened in a succession
dispute between emir Dost Mohammad and former emir Shah Shujah, whom they installed upon
conquering Kabul in August 1839. It took place during the tenure of Lord Auckland. Hence,
statement 4 is incorrect.

12 By which of the following Acts, a law member was added to Governor-General’s Council?

A. Pitt’s India, 1784


B. Charter Act, 1813
C. Charter Act, 1833
D. Charter Act, 1853

Your Answer : C
Correct Answer : C

Answer Justification :

Provision was made under the Charter Act, 1833 for the addition of a law member to the
Governor-General’s Council. Lord Macaulay was the first Law Member appointed under the
Act. The Governor-General was also advised to appoint a Law Commission to codify all the laws.
Hence, option (c) is correct.

13 Who started the policy of Doctrine of Lapse in India?

A. Lord Wellesley
B. Warren Hastings
C. William Bentinck
D. Lord Dalhousie

Your Answer : D
Correct Answer : D

Answer Justification :

The most important tool of Dalhousie’s policy was the Doctrine of Lapse. Under this doctrine
when the ruler of a protected state died without a natural heir, his state was not to pass to an
adopted heir as sanctioned by the age-old tradition of the country. Instead, it was to be annexed to
British India, unless the adoption had been clearly approved earlier by the British authorities. Many
states, including Satara, Nagpur and Jhansi were annexed by applying this doctrine.

Hence, option (d) is correct.

14 Consider the following pairs of peasant movements and the leaders associated with them.
1. Pabna Agrarian Movement – Ishan Chandra Roy
2. Revolt of the Ramosis – Chittur Singh

www.insightsactivelearn.com 8
Online Prelims TEST - 3 (SUBJECT WISE)
( InsightsIAS Mock Test Series for UPSC Preliminary Exam 2020 )

3. Deccan Riots – Rajkumar Shukla

Which of the above pair(s) is/are correctly matched?


A. 1 only
B. 1 and 2 only
C. 2 and 3 only
D. 1, 2 and 3

Your Answer : A
Correct Answer : B

Answer Justification :

In the permanent settlement areas of East Bengal (now in Bangladesh), the Zamindars used various
methods to deprive the ryots of their occupancy rights. The peasants revolted when the Zamindars
increased rent through cesses (tax) and tried to prevent them from acquiring the occupancy rights
under the Act of 1859 through fraudulent measurement, illegal coercion and forced eviction. In May
1874, an Agrarian League was formed.

The storm centre of the movement was Pabna, a comparatively prosperous jute-growing district
where peasants under the Agrarian League protested against the unjust demands of the Zamindars.
The main leader of this Agrarian League was Ishan Chandra Roy, Shambhu Pal and K.
Mullah. Under the banner of this league the peasants of a number of districts of East Bengal
launched a movement between 1870 and 1885. Hence, first pair is correctly matched.

The Ramosis of Maharashtra once served in the inferior ranks of police in the Maratha
administration. After the fall of the Maratha Kingdom, as a result of the Anglo-Maratha Wars, they
reverted back to their lands and had to face conside

In the permanent settlement areas of East Bengal (now in Bangladesh), the Zamindars used various
methods to deprive the ryots of their occupancy rights. The peasants revolted when the Zamindars
increased rent through cesses (tax) and tried to prevent them from acquiring the occupancy rights
under the Act of 1859 through fraudulent measurement, illegal coercion and forced eviction. In May
1874, an Agrarian League was formed.

The storm centre of the movement was Pabna, a comparatively prosperous jute-growing district
where peasants under the Agrarian League protested against the unjust demands of the Zamindars.
The main leader of this Agrarian League was Ishan Chandra Roy, Shambhu Pal and K.
Mullah. Under the banner of this league the peasants of a number of districts of East Bengal
launched a movement between 1870 and 1885. Hence, first pair is correctly matched.

The Ramosis of Maharashtra once served in the inferior ranks of police in the Maratha
administration. After the fall of the Maratha Kingdom, as a result of the Anglo-Maratha Wars, they
reverted back to their lands and had to face considerable distress. Chittur Singh, who revolted in
Satara in 1822 as a protest against heavy assessment, gathered the Ramosis under his banner.
Hence, second pair is correctly matched.

The cultivators of Champaran district in Bihar in the 19th century had been forced by European
planters to cultivate indigo on 3/20th of their holding-known as tinkathia system. With the declining

www.insightsactivelearn.com 9
Online Prelims TEST - 3 (SUBJECT WISE)
( InsightsIAS Mock Test Series for UPSC Preliminary Exam 2020 )

market for indigo in the face of synthetic dyes, the planters were now willing to release the farmers
from their irksome crop but only by recurring enhancements in rent and other illegal dues.
Persuaded by a local man Rajkumar Shukla, in 1917, Mahatma Gandhi offered civil
disobedience. Hence, third pair is not correctly matched.

rable distress. Chittur Singh, who revolted in Satara in 1822 as a protest against heavy
assessment, gathered the Ramosis under his banner. Hence, second pair is correctly matched.

The cultivators of Champaran district in Bihar in the 19th century had been forced by European
planters to cultivate indigo on 3/20th of their holding-known as tinkathia system. With the declining
market for indigo in the face of synthetic dyes, the planters were now willing to release the farmers
from their irksome crop but only by recurring enhancements in rent and other illegal dues.
Persuaded by a local man Rajkumar Shukla, in 1917, Mahatma Gandhi offered civil
disobedience. Hence, third pair is not correctly matched.

15 Which of the following is/are recommendations of the Hunter Education Commission 1882- 83?
1. Primary education should be regarded as education of the masses.
2. Medium of Instruction in primary education should be English.
3. The fees charged in aided secondary schools should be considerably lower than the fees charged in
Government schools.

Select the correct answer using the code given below:


A. 1 and 3 only
B. 2 and 3 only
C. 1 and 2 only
D. 1, 2 and 3

Your Answer : A
Correct Answer : A

Answer Justification :

Lord Ripon appointed the Indian Education Commission on 3rd February 1882, with Sir William
Hunter as its Chairman. It is known as Hunter Commission of 1882.

Major Recommendations of Hunter Commission of 1882 on Primary Education were.

Primary education should be regarded as education of the masses. Hence, statement 1


is correct.
Education should be able to train the people for self-dependence.
Medium of Instruction in primary education should be the mother tongue. Hence,
statement 2 is incorrect.
Appointment of teachers should be made by the district authority and approved by the
government.

Major Recommendations of Hunter Commission on secondary education were

The administrative responsibility on Secondary education should be handed over to the

www.insightsactivelearn.com 10
Online Prelims TEST - 3 (SUBJECT WISE)
( InsightsIAS Mock Test Series for UPSC Preliminary Exam 2020 )

efficient and educated people.


English should remain as medium of instruction in the Secondary stage.
The fees charged in aided secondary schools should be considerably lower than the
fees charged in Government schools. Hence, statement 3 is correct.

16 Young Bengal Movement was started by

A. Raja Ram Mohan Roy


B. David Hare
C. Debendranath Tagore
D. Henry Vivian Derozio

Your Answer : D
Correct Answer : D

Answer Justification :

Young Bengal a socio-intellectual label that was given by the contemporary Calcutta society to the
students of hindu college who followed their teacher Henry Louis Vivian Derozio, a free thinker
and rationalist.

Derozio taught his students to develop critical outlook about life and societal processes. He taught
them how social institutions take root and develop and how people become attached to dead and
fossilised ideas and institutions. Drawing examples from world history and philosophy, Derozio tried
to persuade his students to love knowledge and abandon the habit of believing groundlessly.

Hence, option (d) is correct.

17 The Great Trigonometrical Survey was a project which aimed to measure the entire Indian
subcontinent with scientific precision, was started by?

A. James Prinsep
B. George Everest
C. William Lambton
D. Alexander Cunningham

Your Answer : A
Correct Answer : C

Answer Justification :

The Great Trigonometrical Survey was a project which aimed to measure the entire Indian
subcontinent with scientific precision. It was begun in 1802 by the infantry officer William
Lambton, under the auspices of the East India Company. Under the leadership of his
successor, George Everest, the project was made a responsibility of the Survey of India.

Hence, option (c) is correct.

www.insightsactivelearn.com 11
Online Prelims TEST - 3 (SUBJECT WISE)
( InsightsIAS Mock Test Series for UPSC Preliminary Exam 2020 )

18 With reference to Anglo-Mysore wars, consider the following statements:


1. The first Anglo-Mysore war was ended with Treaty of Madras.
2. Hyder Ali was died in the course of the third Anglo-Mysore war.
3. The third Anglo-Mysore war was fought during the tenure of Governorship of John Shore.

Which of the statements given above is/are correct?


A. 1 only
B. 1 and 2 only
C. 2 and 3 only
D. 1, 2 and 3

Your Answer : A
Correct Answer : A

Answer Justification :

The Second Anglo-Mysore war was fought during Warren Hastings’s tenure. After the first Anglo-
Mysore war the treaty of Madras was signed under which it was agreed that the Company and
Mysore ruler Hyder Ali would help each other in case a third party invaded any one of them.
However, when the Marathas attacked the Mysore, the British did not help. Hyder Ali was further
irritated when British occupied Mahe and Guntur. So he made alliance with the Marathas and the
Nizam of Hyderabad and declared war against the British.

In the midst of the war Hyder Ali died and his son Tipu Sultan continued the war. The
Treaty of Mangalore brought an end to the war. The treaty established friendly relation between the
British and Mysore.

Hence, statement 1 is correct whereas statement 2 is incorrect.

The Third Anglo-Mysore war was fought during the tenure of Lord Cornwallis against Tipu Sultan.
The British defeated Tipu and signed the Treaty of Seringapatam. By this treaty half of the
dominion of Tipu was taken away and divided among the British, the Marathas and the Nizam.

Hence, statement 3 is incorrect.

19 With reference to the Anglo-Maratha wars, consider the following statements:


1. As per the Treaty of Salbai which was signed after the end of First Anglo –Maratha was, Salsette
should continue in the possession of the English.
2. As per the Treaty of Bassein which was signed after the end Second Anglo-Maratha war, Peshwa
agreed to give up all claims for Chauth on the Nizam’s dominions.

Which of the statements given above is/are correct?


A. 1 only
B. 2 only
C. Both 1 and 2
D. Neither 1 nor 2

Your Answer : C
www.insightsactivelearn.com 12
Online Prelims TEST - 3 (SUBJECT WISE)
( InsightsIAS Mock Test Series for UPSC Preliminary Exam 2020 )

Correct Answer : C

Answer Justification :

The Treaty of Salbai was signed on 17 May 1782, by representatives of the Maratha Empire and
the British East India Company after long negotiations to settle the outcome of the First Anglo-
Maratha War.

As per the provision of Treaty of Salbai, Salsette should continue in the possession of the English.
Hence, statement 1 is correct.

The Treaty of Bassein was a pact signed on 31 December 1802 between the British East India
Company and Baji Rao II, the Maratha Peshwa of Pune in India after the Battle of Poona. As per the
provision of Treaty of Bassein, Peshwa agreed to give up all claims for Chauth on the Nizam’s
dominions. Hence, statement 2 is correct.

20 Bakshi Jagabandhu, a famous freedom fighter is associated with

A. Kutch Rebellion
B. Waghera Rising
C. Paika Rebellion
D. Poligars’ Revolt

Your Answer : C
Correct Answer : C

Answer Justification :

Jagabandhu Bidyadhara Mohapatra Bhramarbara Rai popularly known as "Bakshi Jagabandhu" or


"Paika Bakshi" was the commander of the forces of the king of Khordha. He is one of the
earliest freedom fighters of India. The great Paika rebellion in 1817 was under his leadership.

Hence, option (c) is correct.

21 With reference to the policy of Subsidiary Alliance, consider the following statements:
1. Those native princes or rulers who would enter into Subsidiary Alliance would not be free to declare
war against any other power without the consent of the English.
2. The doctrine of subsidiary alliance was introduced by Lord Dalhousie.

Which of the statements given above is/are correct?


A. 1 only
B. 2 only
C. Both 1 and 2
D. Neither 1 nor 2

Your Answer : A
Correct Answer : A

www.insightsactivelearn.com 13
Online Prelims TEST - 3 (SUBJECT WISE)
( InsightsIAS Mock Test Series for UPSC Preliminary Exam 2020 )

Answer Justification :

Wellesley’s aims were expansionism and imperialism. Wellesley naturally followed a policy of
unabashed imperialism and taking advantage of the eagerness of the native princes and powers to
enlist British support in their mutual power-struggle, he wanted to make them absolutely dependent
on the British. This policy goes by the name Subsidiary Alliance. Thus, statement 2 is incorrect.

Those native princes or rulers who would enter into Subsidiary Alliance would not be free to declare
war against any other power, nor enter into negotiations with, any power, native or otherwise
without the consent of the English,

The princes who were comparatively strong and powerful would be permitted to retain their armies,
but their armies must be placed under British generals.

Hence, statement 1 is correct.

22 Consider the following statements:


1. James Augustus Hicky in 1780 started the newspaper Bombay Samachar.
2. The Lahore Chronicle was started by Syed Muhammad Azim.

Which of the statements given above is/are correct?


A. 1 only
B. 2 only
C. Both 1 and 2
D. Neither 1 nor 2

Your Answer :
Correct Answer : B

Answer Justification :

James Augustus Hicky was an Irishman who launched the first printed newspaper in India,
Hicky's Bengal Gazette. The Bombay Samachar, now Mumbai Samachar, is the oldest
continuously published newspaper in India. Established in 1822 by Fardunjee Marzban, it is
published in Gujarati and English. Hence, statement 1 is incorrect.

The Lahore Chronicle was started by Syed Muhammad Azim, father of the Punjab historian, in 1849.
Hence, statement 2 is correct.

23 Consider the following statements:


1. Warren Hastings established Calcutta Madrasa was founded for the study of Muslim law and related
subjects.
2. Jonathan Duncan, started a Sanskrit College at Varanasi for study of Hindu law and Philosophy.

Which of the statements given above is/are correct?


A. 1 only
B. 2 only

www.insightsactivelearn.com 14
Online Prelims TEST - 3 (SUBJECT WISE)
( InsightsIAS Mock Test Series for UPSC Preliminary Exam 2020 )

C. Both 1 and 2
D. Neither 1 nor 2

Your Answer : C
Correct Answer : C

Answer Justification :

Calcutta Madrasah, the first educational institution was set up in the Indian subcontinent
by Warren Hastings, the then Governor General of East India Company in 1870. It became a
leading centre of Islamic and Muslims' education. Hence, statement 1 is correct.

Jonathan Duncan was Governor of Bombay from 27 December 1795 until his death in 1811. In
1791, he started the Sanskrit College at Benares for the study of Hindu law and Philosophy. (In
1958 the Sanskrit College became a university and in 1974 the name was changed
to Sampurnanand Sanskrit University.) Hence, statement 2 is correct.

24 Which of the following acts led to the legislative centralization in the administration of India?

A. Charter Act, 1833


B. Charter Act, 1813
C. Charter Act, 1853
D. Government of India Act, 1858

Your Answer : D
Correct Answer : A

Answer Justification :

The Act of 1833 for the first time initiated the creation of a strong centralized government in India.
With legislative centralization it ensured uniformity of laws in the country. Above all it
ensured a fair and impartial treatment to Indians in matter of selection for State service. All
variations and complexities in legal system from the Province were removed. All the rules and
regulations were codified. The Indian Penal Code and the Code of Civil and Criminal
Procedure came into existence. Hence, option (a) is correct.

25 With reference to Lex Loci Act of 1850, which of the following statements is correct?

A. It provided tax concession to British traders.


B. It extended the prohibition of Sati across all over British India.
C. It provided right to inherit ancestral property to Hindu converts to Christianity.
D. It granted the legal recognition to widow’s marriage.

Your Answer :
Correct Answer : C

Answer Justification :

www.insightsactivelearn.com 15
Online Prelims TEST - 3 (SUBJECT WISE)
( InsightsIAS Mock Test Series for UPSC Preliminary Exam 2020 )

Lex Loci Act was proposed in 1845 and passed in 1850, which provided the right to inherit ancestral
property to Hindu converts to Christianity.

Hence, option(c) is correct.

26 In the context of freedom struggle during 19th century, the Azamgarh proclamation was associated
with?

A. Formation of Indian National Congress


B. Revolt of 1857
C. Vellore Mutiny
D. Second Anglo-Maratha War

Your Answer : C
Correct Answer : B

Answer Justification :

The Azamgarh proclamation was published in the Delhi Gazette in the midst of the “Great Mutiny”
of 1857. The author was most probably Firoz Shah, a grandson of the Mughal emperor Bahadur
Shah Zafar, whose restoration to full power was a main aim of the rebels. Hence, option (b) is
correct.

27 Who of the following social reformers published a set of notes on “Infant Marriage and Enforced
Widowhood in India”?

A. Ishwarchand Vidyasagar
B. Raja Rammohun Roy
C. B. M. Malabari
D. Swami Dayananada Saraswati

Your Answer : A
Correct Answer : C

Answer Justification :

Behramji Merwanji Malabari was an Indian poet, publicist, author, and social reformer best known
for his ardent advocacy for the protection of the rights of women and for his activities against child
marriage. He published a set of notes on “Infant Marriage and Enforced Widowhood in India”.
Hence, option (c) is correct.

28 With reference to Deccan Riots in the late 19th century, consider the following statements:
1. It was mainly directed against the British’s oppressive land revenue policies.
2. The Agriculturists Relief Act of 1879 was passed to provide relief to the Deccan peasants.

Which of the statements given above is/are correct?

www.insightsactivelearn.com 16
Online Prelims TEST - 3 (SUBJECT WISE)
( InsightsIAS Mock Test Series for UPSC Preliminary Exam 2020 )

A. 1 only
B. 2 only
C. Both 1 and 2
D. Neither 1 nor 2

Your Answer : C
Correct Answer : B

Answer Justification :

In the Ryotwari areas of the Deccan region, especially in the districts of Pune and Ahmednagar the
land revenue was very high. Yet, it was periodically enhanced. The revenue demand of the British
was required to be paid even when a succession of bad harvest followed. In these circumstances the
peasants had nowhere to turn except money-lenders, must of whom were outsider Marwaris or
Gujaratis. The latter exploited this situation to their advantage and obtained mortgage of a very
large portion of land against rural loans. The situation became more and more explosive as the
peasants were faced with the brutal alternative of mortgaging land or eviction.

The Maratha peasants finally gave vent to their feelings and started social boycott of the
moneylenders in Poona and Ahmednagar. They forcibly seized the debt bonds and set them on fire.
The riots began in Sirur taluk in December 1874 and by September 1875; the Deccan riots affected
33 places in six taluks. Hence, statement 1 is incorrect.

The uprising was completely crushed by the police assisted by the military, and nearly one thousand
peasants were arrested. The Government of India also appointed a commission to inquire
into the nature and cause of these riots. The Commission unanimously held that poverty and
consequent indebtedness of the cultivators were the real causes of the riots. Following the
Commission’s recommendations, the Government passed the Agriculturists Relief Act
1879, which imposed restriction on alienation of peasants’ lands and on the Operations of the Civil
Procedure Code. Hence statement 2 is correct.

29 Nil-Darpan, a play written by Din Bandhu Mitra, portrays the plight of

A. Higher caste women in Northwest India


B. Indigo Planters
C. Child labours in European Industries
D. Oppression of tribals in central India.

Your Answer : B
Correct Answer : B

Answer Justification :

The Indigo Rebellion (Neel Bidroho) took place in Bengal in 1859-60 and was a revolt by the
farmers against British planters who had forced them to grow indigo under terms that were greatly
unfavorable to the farmers.

The play Nil Darpan (The Mirror of Indigo) by Dinabandhu Mitra written in 1858 – 59 portrayed

www.insightsactivelearn.com 17
Online Prelims TEST - 3 (SUBJECT WISE)
( InsightsIAS Mock Test Series for UPSC Preliminary Exam 2020 )

the farmers’ situation accurately. It showed how farmers were coerced into planting indigo
without adequate payment. The play became a talking point and it urged the Bengali
intelligentsia to lend support to the indigo revolt. Hence option (b) is correct.

30 Consider the following statements.


1. The Dutch established their first factory at Surat in 1605.
2. Sultan of Bahamani issued the Golden Farman to the English Company in 1632.

Which of the statements given above is/are correct?


A. 1 only
B. 2 only
C. Both 1 and 2
D. Neither 1 nor 2

Your Answer : A
Correct Answer : D

Answer Justification :

The Dutch East India Company was created in 1602 as “United East India Company” and its first
permanent trading post was in Indonesia. In India, they established the first factory in
Masulipattanam in 1605, followed by Pulicat in 1610, Surat in 1616, Bimilipatam in 1641 and
Chinsura in 1653. Hence statement 1 is incorrect.

In 1615, a British mission under Sir Thomas Roe succeeded in obtaining farmans from the Mughal
Court confirming free trade without liability to pay inland toll. In 1632 the English obtained
from the Sultan of Golconda the Golden farman granting them the right to trade throughout
the kingdom of Golconda on payment of a fixed customs duty of 500 pagodas per year. Hence
statement 2 is incorrect.

31 Consider the following statements:


1. Mysore was annexed by William Bentinck on the grounds of misgovernance
2. Dupleix led the French army in the Third Carnatic War.

Which of the statements given above is/are correct?


A. 1 only
B. 2 only
C. Both 1 and 2
D. Neither 1 nor 2

Your Answer : B
Correct Answer : A

Answer Justification :

Mysore in was annexed by William Bentinck in1831 on the ground of misgovernment; it was not
annexed, but it was administered on behalf of the raja for the next 50 years. Hence, statement 1 is

www.insightsactivelearn.com 18
Online Prelims TEST - 3 (SUBJECT WISE)
( InsightsIAS Mock Test Series for UPSC Preliminary Exam 2020 )

correct.

The Third Carnatic war was a local version of the Seven Years war in Europe. The Third Carnatic
War put an end to the French ambitions to create a colonial empire in India. The British Forces
were able to capture the French Settlements at Chandranagar in 1757. The French forces in
south were led by Conte De Lally. The British forces under Sir Eyre Coote, defeated the
French in the Battle of Wandiwash in 1760 and besieged Pondicherry. Hence, statement 2
is incorrect.

32 Which of the following was/were part of pre-congress campaign against British?


1. Against Lytton’s Afghan adventure
2. Against the Partition of Bengal
3. Against the Ilbert Bill.

Select the correct answer using the codes given below.


A. 1 and 3 only
B. 1 only
C. 2 and 3 only
D. 3 only

Your Answer : B
Correct Answer : A

Answer Justification :

Major pre-congress campaign were

For imposition of import duty on cotton (1875)

For Indianisation of government service (1878-79)

Against Lytton’s Afghan adventure (Hence, Statement 1 is correct)

Against Arms Act (1878)

Against Vernacular Press Act (1878)

For right to join volunteer corps

Against plantation labour and against Inland Emigration Act

In support of Ilbert Bill (not against the bill. Hence, statement 3 is incorrect)

www.insightsactivelearn.com 19
Online Prelims TEST - 3 (SUBJECT WISE)
( InsightsIAS Mock Test Series for UPSC Preliminary Exam 2020 )

For an All India Fund for Political Agitation

Campaign in Britain to vote for pro-India party

Protest against the Partition of Bengal(1905) was a post congress campaign. Hence,
statement 2 is incorrect.

33 Consider the following statements


1. Job Charnock established Fort William at Calcutta
2. Francis Day established the city of Calcutta.

Which of the statements given above is/are correct?


A. 1 only
B. 2 only
C. Both 1 and 2
D. Neither 1 nor 2

Your Answer : C
Correct Answer : A

Answer Justification :

By the year 1690, Job Charnock, the agent of the East India Company purchased three villages
namely, Sutanuti, Govindpur and Kalikatta, which, in course of time, grew into the city of Calcutta.
It was fortified byJob Charnock, who named it Fort William after the English King,William III.
Hence, statement 1 is correct.

In 1639, Francis Day established the city of Madras and constructed the Fort St. George.
Hence, statement 2 is incorrect.

34 Consider the following statements:


1. Battle of Plassey was fought during reign of Mughal ruler Shah Alam II.
2. Akbar II gave the title of Raja to Ram Mohan Roy.

Which of the statements given above is/are correct?


A. 1 only
B. 2 only
C. Both 1 and 2
D. Neither 1 nor 2

Your Answer : C
Correct Answer : B

Answer Justification :

www.insightsactivelearn.com 20
Online Prelims TEST - 3 (SUBJECT WISE)
( InsightsIAS Mock Test Series for UPSC Preliminary Exam 2020 )

During the reign of Alamgir II(1754-1758), battle of Plassey was fought in June 1757. During the
reign of Shah Alam II(1759-1806), two decisive battles- the Third battle of Panipat (1761) and
the Battle of Buxar(1764) were fought.

Hence, statement 1 is incorrect.

Akbar II (1806-37) gave the title of Raja to Rammohan Roy. Hence, statement 2 is correct.

35 In 1720, the British government enacted the Calico Act. What is it related to?

A. Disallowing Indians from entering Civil Services


B. Compulsory use of English language in Company affairs
C. Banning the use of printed cotton textile, imported from India.
D. Barring of company officials from private trade

Your Answer :
Correct Answer : C

Answer Justification :

In 1720, the British government enacted a legislation banning the use of printed cotton textiles –
chintz – in England. Interestingly, this Act was known as the Calico Act.

At this time textile industries had just begun to develop in England. Unable to compete with Indian
textiles, English producers wanted a secure market within the country by preventing the entry of
Indian textiles. The first to grow under government protection was the calico printing industry.
Indian designs were now imitated and printed in England on white muslin or plain unbleached
Indian cloth. Hence, option (c) is correct.

36 Who among the following European powers were the last to come to India as traders before Indian
Independence?

A. Dutch
B. English
C. French
D. Portuguese

Your Answer : A
Correct Answer : C

Answer Justification :

It was the Portuguese who first discovered a direct sea route to India. Portuguese sailor Vasco da
Gama arrived at Calicut an important sea port located on the South-West India on May 20, 1498
AD.

Next to the Portuguese, the Dutch set their feet in India. The Dutch founded their first factory in

www.insightsactivelearn.com 21
Online Prelims TEST - 3 (SUBJECT WISE)
( InsightsIAS Mock Test Series for UPSC Preliminary Exam 2020 )

Masaulipatam in Andhra Pradesh in 1605. Subsequently they also established trading centres in
various parts of India.

In 1608 AD, the East India Company sent Captain William Hawkins to the court of the
Mughal emperor Jahangir to secure royal patronage. He succeeded in getting royal permit for the
Company to establish its factories at various places on the Western coast of India.

The last European people to arrive in India were the French. The French East India Company
was formed in 1664 AD during the reign of King Louis XIV to trade with India. In 1668 AD the
French established their first factory at Surat and in 1669 AD established another French
factory at Masaulipatam.

Hence, option(c) is correct.

37 Consider the following Princely States:


1. Hyderabad
2. Mysore
3. Punjab

Which of the above given princely states accepted Subsidiary Alliance system?
A. 1 and 2 only
B. 1 only
C. 2 and 3 only
D. 1, 2 and 3

Your Answer : D
Correct Answer : A

Answer Justification :

The Indian princes who accepted the subsidiary system were: the Nizam of Hyderabad (1798 and
1800), the ruler of Mysore(1799), the ruler of Tanjore(1799), the Nawab of Awadh etc.
Hence statements 1 and 2 are correct.

Punjab was annexed by Dalhousie by means of war. Hence, statement 3 is incorrect.

38 The infamous Black Hole Tragedy is associated with:

A. Battle of Panipat
B. Battle of Plassey
C. Battle of Buxar
D. Battle of Karnal

Your Answer : C
Correct Answer : B

Answer Justification :

www.insightsactivelearn.com 22
Online Prelims TEST - 3 (SUBJECT WISE)
( InsightsIAS Mock Test Series for UPSC Preliminary Exam 2020 )

The Black Hole of Calcutta was a small prison or dungeon in Fort William where troops of Siraj ud-
Daulah, the Nawab of Bengal, held British prisoners of war for three days on 20 June 1756. This
infamous incident led to Battle of Plassey.

Hence, option (b) is correct.

39 Which of the following was/were the attributes of the Indigo Revolt 1859-60?
1. The peasants were supported by the intelligentsia and press.
2. It was led by Degambar and Bishnu Biswas.

Select the correct answer using the code given below


A. 1 only
B. 2 only
C. Both 1 and 2
D. Neither 1 nor 2

Your Answer : C
Correct Answer : C

Answer Justification :

The peasants were supported by the intelligentsia and press. The play Nil Darpan (The Mirror of
Indigo) by Dinabandhu Mitra written in 1858 – 59 portrayed the farmers’ situation accurately.
It showed how farmers were coerced into planting indigo without adequate payment. The
play became a talking point and it urged the Bengali intelligentsia to lend support to the indigo
revolt. Hence, statement 1 is correct.

The Indigo revolt was a peasant movement and subsequent uprising of indigo farmers against the
indigo planters that arose in Bengal in 1859. It was led by Degambar and Bishnu Biswas. Hence
statement 2 is correct.

40 Continental system by Napoleon led to which of the following reform in India?

A. Regulating Act, 1773


B. Charter Act, 1813
C. Charter Act, 1833
D. Charter Act, 1793

Your Answer :
Correct Answer : B

Answer Justification :

In England, the business interests were pressing for an end to the Company's monopoly over trade
in India because of a spirit of laissezfaire and the continental system by Napoleon by which the
European ports were closed to Britain. The 1813 Act sought to redress these grievances. Hence
option (b) is correct.

www.insightsactivelearn.com 23
Online Prelims TEST - 3 (SUBJECT WISE)
( InsightsIAS Mock Test Series for UPSC Preliminary Exam 2020 )

41 With reference to the Pitt’s India Act of 1784, consider the following statements:
1. The presidencies of Bombay and Madras were made subordinate to the governor-general.
2. A Board of Control consisting of the chancellor of exchequer, a secretary of state and four members
of the Privy Council were to exercise control over the Company's civil, military, and revenue affairs

Which of the statements given above is/are correct?


A. 1 only
B. 2 only
C. Both 1 and 2
D. Neither 1 nor 2

Your Answer : C
Correct Answer : C

Answer Justification :

Pitt's India Act of 1784:

In India, the governor-general was to have a council of three (including the commander-in-chief),
and the presidencies of Bombay and Madras were Made subordinate to the governor-general.
Hence, statement 1 is correct.

The Government's control over the Company's affairs was greatly extended. A Board of Control
consisting of the chancellor of exchequer, a secretary of state and four members of the Privy
Council (to be appointed by the Crown) were to exercise control over the Company's civil, military,
and revenue affairs. All dispatches were to be approved by the board. Thus a dual system of control
was set up. Hence statement 2 is correct.

42 Who wrote the Biography of A.O. Hume?

A. Webb Miller
B. William Jones
C. William Wedderburn
D. None of the above

Your Answer : B
Correct Answer : C

Answer Justification :

Sir William Wedderburn was a Scottish civil servant in India and a politician. He wrote a
biographical memoir of A. O. Hume who died in 1912. Hence, option(c) is correct.

43 Which European Company first started the strategy of using its well-equipped army to intervene in
the mutual quarrels of Indian states?

A. Portuguese

www.insightsactivelearn.com 24
Online Prelims TEST - 3 (SUBJECT WISE)
( InsightsIAS Mock Test Series for UPSC Preliminary Exam 2020 )

B. French
C. English
D. Dutch

Your Answer : A
Correct Answer : C

Answer Justification :

Dupleix, the French Governor-General at Pondicherry, evolved the strategy of using the well
disciplined, modern French army to intervene in the mutual quarrels of the Indian princes and by
supporting one against the other, securing monetary, commercial or territorial favours from the
victor.

Hence, option (b) is correct.

44 Consider the following statements:


1. In 1835, Governor-General Bentinck passed a resolution regarding the promotion of European
literature and Sciences through the medium of vernacular languages.
2. Bentinck favoured Macaulay’s Report (1835) on English education but did not get support from the
Court of Directors.

Which of the statements given above is/are correct?


A. 1 only
B. 2 only
C. Both 1 and 2
D. Neither 1 nor 2

Your Answer :
Correct Answer : D

Answer Justification :

The government of William Bentinck in the Resolution in March 1835 accepted the viewpoint of
Macaulay that in future, the object of the company’s government should be the promotion of
European literature and sciences, through the medium of English.

Hence, both statements are incorrect.

45 Consider the following statements about the effects of Land Revenue Systems introduced by the
British:
1. Over-assessment to maximise revenue.
2. Arrears of payments, hence mounting debts on peasants.
3. Land dispossession
4. Rising agrarian disturbances.

Which of the statements given above is/are correct?

www.insightsactivelearn.com 25
Online Prelims TEST - 3 (SUBJECT WISE)
( InsightsIAS Mock Test Series for UPSC Preliminary Exam 2020 )

A. 1 and 2 only
B. 2 and 3 only
C. 1, 2 and 3 only
D. 1, 2, 3 and 4

Your Answer : D
Correct Answer : D

Answer Justification :

All the statements given above are the effects of Land Revenue Systems introduced by the British.

46 During the rule of East India Company which Indian state was annexed on ground of misrule?

A. Punjab
B. Jhansi
C. Awadh
D. Hyderabad

Your Answer : C
Correct Answer : C

Answer Justification :

In 1856, Dalhousie annexed Awadh on the ground of Good of the Governed i.e. misrule of Nawab
Wajid Ali Shah. Hence, option(c) is correct.

47 In the context of East India Company, the term ‘Dastak’ was

A. free transit pass with payment of custom dues at tolls.


B. free transit pass without payment of custom dues at tolls.
C. free transit pass for the company’s trade and private trade of Indian merchants.
D. free transit pass issued by the company for Indian merchants in Calcutta.

Your Answer : B
Correct Answer : B

Answer Justification :

Company got Royal firmans from Mughal emperors. The firmans allowed the company to carry out
duty free trade by paying fixed amount in lieu of trade. They were allowed free transit at toll
chowkies. This was done through free transit passes called Dastak.

Hence, option (b) is correct.

48 Consider the following statements:

www.insightsactivelearn.com 26
Online Prelims TEST - 3 (SUBJECT WISE)
( InsightsIAS Mock Test Series for UPSC Preliminary Exam 2020 )

1. After 1805, Wellesley was recalled from India.


2. The British government wanted to use military generalship of Wellesley against the French threat in
Europe.

Which of the statements given above is/are correct?


A. 1 only
B. 2 only
C. Both 1 and 2
D. Neither 1 nor 2

Your Answer : C
Correct Answer : A

Answer Justification :

The shareholders of the East India Company discovered that the policy of expansion through war
was proving costly and was reducing their profits. The Company’s debt had increased from ₤17
million in 1797 to ₤31 million in 1806. Moreover, Britain’s finances were getting exhausted at a
time when Napoleon was once again becoming a major threat in Europe. British statesmen and the
directors of the Company felt that time had come to check further expansion, to put an end to
ruinous expenditure, and to digest and consolidate Britain’s recent gains in India. Wellesley was,
therefore, recalled from India and the Company made peace with Holkar in January 1806 by the
treaty of Raighat, giving back to the Holkar the greater part of his territories.

Hence, statement 1 is correct. However, statement 2 is not correct.

49 With reference to Wood’s Despatch, consider the following statements:


1. It asked the Government of India to assume responsibility for education of the masses.
2. It recommended mother tongue as the medium of instruction for higher studies and vernaculars at
school level.

Which of the statements given above is/are correct?


A. 1 only
B. 2 only
C. Both 1 and 2
D. Neither 1 nor 2

Your Answer : C
Correct Answer : A

Answer Justification :

In 1854, the Court of Directors of the East India Company in London sent an educational despatch
to the Governor-General in India. Issued by Charles Wood, the President of the Board of Control of
the Company, it has come to be known as Wood’s Despatch.

It asked the Government of India to assume responsibility for education of the


masses,thus repudiating the 'downward filtration theory', at least on paper. Hence, statement 1 is
www.insightsactivelearn.com 27
Online Prelims TEST - 3 (SUBJECT WISE)
( InsightsIAS Mock Test Series for UPSC Preliminary Exam 2020 )

correct.

It recommended English as the medium of instruction for higher studies and vernaculars at
school level. Hence, statement 2 is incorrect.

It laid stress on female and vocational, education, and on teachers' training. It laid down that the
education imparted in government institutions should be secular. It recommended a system of
grants-in-aid to encourage private enterprise.

50 Which of the following Governors General/Viceroys supported the cause of Indian Press?
1. Wellesley
2. Metcalfe
3. Lytton
4. Ripon

Select the correct answer using the code given below:


A. 1 and 2 only
B. 1, 2 and 3 only
C. 2 and 4 only
D. 1, 2 and 4 only

Your Answer : C
Correct Answer : C

Answer Justification :

In 1835, Metcalfe had lifted restrictions imposed on the Indian press. Hence, he is known as the
Liberator of Indian Press.

In 1882 Ripon also lifted the restrictions on Indian Press earlier imposed by the Vernacular
Press Act, 1878 imposed by Lytton.

Hence, option(c) is correct.

51 Consider the following tribal movements/uprisings


1. Pahariyas’ Rebellion
2. Kol Mutiny
3. Ho and Munda Uprisings
4. Khond Uprisings

Arrange the above events in chronological order


A. 2-3-1-4
B. 4-1-2-3
C. 1-3-2-4
D. 1-2-3-4

Your Answer :

www.insightsactivelearn.com 28
Online Prelims TEST - 3 (SUBJECT WISE)
( InsightsIAS Mock Test Series for UPSC Preliminary Exam 2020 )

Correct Answer : C

Answer Justification :

Pahariyas’ Rebellion: The British expansion on their territory led to an uprising by the martial
Pahariyas of the Raj Mahal Hills in 1778

Ho and Munda Uprisings (1820-1837)

Kol Mutiny (1831)

Khond Uprisings (1837-1856)

52 Daman-i-koh, in modern Indian history is known for

A. It is a region where Santhal rebellion took place against the British


B. It is a region where the Mughals established their supremacy over Eastern India
C. It is a region where the British executed last living Mughal lineage
D. None of the above

Your Answer : A
Correct Answer : A

Answer Justification :

Among the numerous tribal revolts, the Santhal hool or uprising was the most massive. The
Santhals, who live in the area between Bhagalpur and Rajmahal, known as Daman-i-koh, rose in
revolt; made a determined attempt to expel the outsiders — the dikus — and proclaimed the
complete annihilation of the alien regime.

The Damin-i-koh was a densely forested and hilly area. Even in the valleys there was hardly any
human interference except for an occasional Paharia village.

53 Consider the following statements


1. Bhagat Jawahar Mal led a sundry army of Paikas against East India Company.
2. The Kuka Movement was founded in 1840 by Bakshi Jagabandhu Bidyadhar in western Punjab.

Which of the statements given above is/are correct?


A. 1 only
B. 2 only
C. Both 1 and 2
D. Neither 1 nor 2

Your Answer : C
Correct Answer : D

Answer Justification :

www.insightsactivelearn.com 29
Online Prelims TEST - 3 (SUBJECT WISE)
( InsightsIAS Mock Test Series for UPSC Preliminary Exam 2020 )

Statement 1 is INCORRECT: With active support of Mukunda Deva, the last Raja of Khurda, and
other zamindars of the region, Bakshi Jagabandhu Bidyadhar led a sundry army of Paikas
forcing the East India Company forces to retreat for a time. The rebellion came to be known as the
Paika Bidroh (rebellion).

Statement 2 is INCORRECT: The Kuka Movement was founded in 1840 by Bhagat Jawahar Mal
(also called Sian Saheb) in western Punjab. A major leader of the movement after him was Baba
Ram Singh.

54 Which of the following was/were the causes for the1857 revolt?


1. Destroy of traditional economic fabric of the Indian society.
2. Subsidiary Alliance and Doctrine of Lapse
3. Attempts at socio-religious reform by British government

Select the correct answer using the code given below


A. 1 and 2 only
B. 2 only
C. 2 and 3 only
D. 1, 2 and 3

Your Answer : D
Correct Answer : D

Answer Justification :

All the statements are correct

The colonial policies of the East India Company destroyed the traditional economic fabric of
the Indian society. The peasantry were never really to recover from the disabilities imposed by
the new and a highly unpopular revenue settlement.

The East India Company’s greedy policy of aggrandizement accompanied by broken pledges and
promises resulted in contempt for the Company and loss of political prestige, besides causing
suspicion in the minds of almost all the ruling princes in India, through such policies as of
‘Effective Control’, ‘Subsidiary Alliance’ and ‘Doctrine of Lapse’. The right of succession was
denied to Hindu princes.

The attempts at socio-religious reform such as abolition of sati, support to widow-marriage


and women’s education were seen by a large section of the population as interference in the
social and religious domains of Indian society by outsiders.

55 Consider the following events


1. Queen’s Proclamation
2. Mangal Pandey refused to use the greased cartridges and single-handedly attacked and killed his
officer.
3. The city of Delhi fell into the hands of the rebellious soldiers.

www.insightsactivelearn.com 30
Online Prelims TEST - 3 (SUBJECT WISE)
( InsightsIAS Mock Test Series for UPSC Preliminary Exam 2020 )

Arrange the above given events in chronological order


A. 3-2-1
B. 1-2-3
C. 3-1-2
D. 2-3-1

Your Answer : D
Correct Answer : D

Answer Justification :

The events that led to the Revolt began on 29 March 1857 at Barrackpore. Mangal Pandey
(a sepoy) refused to use the greased cartridges and single-handedly attacked and killed his officer.
Mangal Pandey was hanged.

The city of Delhi fell into the hands of the rebellious soldiers on 12 May 1857. Lieutenant
Willtashby, the officer in charge of Delhi could not prevent the mutineers.

Queen Victoria's Proclamation – November 1, 1858. On November 1, 1858, a grand Darbar


was held at Allahabad. Here Lord Canning sent forth the royal proclamation which announced that
the queen had assumed the government of India.

56 Consider the following pairs with respect to


First War of Independence (1857)
List I List II
Nana Saheb Kanpur
Bihar Begum of Oudh
Delhi Kunwar Singh

Which of the pairs given above is/are matched correctly?


A. 1 and 2 only
B. 2 and 3 only
C. 3 only
D. 1 only

Your Answer : D
Correct Answer : D

Answer Justification :

The leadership at Delhi was nominally in the hands of Bahadur Shah, but the real control was
exercised by General Bakht Khan.

At Kanpur the revolt was led by Nana Saheb, the adopted son of Baji Rao II, the last Peshwa.
Nana Saheb expelled the English from Kanpur with the help of the sepoys and proclaimed himself
the Peshwa.

www.insightsactivelearn.com 31
Online Prelims TEST - 3 (SUBJECT WISE)
( InsightsIAS Mock Test Series for UPSC Preliminary Exam 2020 )

Kunwar Singh, a ruined and discontented zamindar of Jagdishpur near Oudh, was the chief
organiser of the revolt in Bihar.

57 Which of the following was/were reasons for the failure of 1857 Revolt
1. Poor arms and equipment
2. Moneylenders, merchants and modern educated Indians were against the revolt
3. British were aided by new scientific inventions

Select the correct answer using the code given below


A. 1 and 2 only
B. 2 and 3 only
C. 1 and 3 only
D. 1, 2 and 3

Your Answer : D
Correct Answer : D

Answer Justification :

All were reasons for the failure of 1857 revolt.

The Indian soldiers were poorly equipped materially, fighting generally with swords and
spears and very few guns and muskets. On the other hand, the European soldiers were equipped
with the latest weapons of war like the Enfield rifle.

Different sections of society such as moneylenders, merchants and modern educated


Indians were actually against the Revolt.

British possessed better equipment. In addition, the British were aided by new scientific
inventions such as the telegraph system and postal communications. This enabled the British to
keep in touch with all parts of the country and to manoeuvre their troops according to their needs

58 Consider the following statements regarding Government of India Act of 1858


1. It changed the designation of the Governor-General of India to that of Viceroy of India.
2. It established a 15-member Council of India to assist the secretary of state for India.
3. It empowered the Viceroy to make rules and orders for the more convenient trans-action of business
in the council.

Which of the statements given above is/are correct?


A. 1 and 2 only
B. 2 and 3 only
C. 1 and 3 only
D. 1, 2 and 3

Your Answer : C
Correct Answer : A

www.insightsactivelearn.com 32
Online Prelims TEST - 3 (SUBJECT WISE)
( InsightsIAS Mock Test Series for UPSC Preliminary Exam 2020 )

Answer Justification :

Government of India Act of 1858

This significant Act was enacted in the wake of the Revolt of 1857—also known as the First War of
Independence or the ‘sepoy mutiny’. The act known as the Act for the Good Government of
India, abolished the East India Company, and transferred the powers of government, territories and
revenues to the British Crown.

Features of the Act

Statement 1 is correct: It provided that India henceforth was to be governed by, and in the name
of, Her Majesty. It changed the designation of the Governor-General of India to that of Viceroy of
India. He (viceroy) was the direct representative of the British Crown in India. Lord Canning thus
became the first Viceroy of India.

Statement 2 is correct: It established a 15-member Council of India to assist the secretary of state
for India. The council was an advisory body. The secretary of state was made the chairman of the
council.

Statement 3 is INCORRECT: Indian Councils Act of 1861 empowered the Viceroy to make rules
and orders for the more convenient trans-action of business in the council.

59 Consider the following statements regarding Lord Lytton


1. First Famine Commission was constituted under Sir Richard Strachey during Lytton time.
2. First Afghan War was took place during Lytton’s time.
3. He abolished many import duties and supported the Free Trade Policy.

Which of the statements given above is/are correct?


A. 1, 2 and 3
B. 1 only
C. 1 and 2 only
D. 1 and 3 only

Your Answer : C
Correct Answer : D

Answer Justification :

Statement 1 is correct: The outbreak of cholera and fever added to the misery of the suffering
population. Lytton’s Government failed miserably to tackle the situation. The government’s relief
measures seemed to be inadequate. The first Famine Commission (1878-80) under Sir
Richard Strachey was appointed and it made many commendable recommendations.

Statement 2 is INCORRECT: Second Afghan War (1878-80) took place during Lord Lytton.

Statement 3 is correct: Lord Lytton introduced uniform salt tax throughout British India. He also
abolished many import duties and supported the Free Trade Policy.

www.insightsactivelearn.com 33
Online Prelims TEST - 3 (SUBJECT WISE)
( InsightsIAS Mock Test Series for UPSC Preliminary Exam 2020 )

60 Consider the following statements regarding Lord Ripon


1. He increased the power of municipalities.
2. Ripon appointed William Hunter commission to improve the prospects of education.
3. He was responsible for the rendition of Mysore to its Hindu ruler.

Which of the statements given above is/are correct?


A. 1 and 2 only
B. 1, 2 and 3
C. 1 and 3 only
D. 3 only

Your Answer : A
Correct Answer : B

Answer Justification :

All statements are correct

Ripon believed that self-government is the highest and noblest principles of politics. Therefore,
Ripon helped the growth of local bodies like the Municipal Committees in towns and the local
boards in taluks and villages. The powers of municipalities were increased.

Ripon wanted to review the working of the educational system on the basis of the recommendations
of the Wood’s Despatch. For further improvement of the system Ripon appointed a Commission
in 1882 under the chairmanship of Sir William Hunter. The Commission came to be known as
the Hunter Commission.

He was also responsible for the rendition of Mysore to its Hindu ruler. Moreover, he
repealed the Vernacular Press Act and earned much popularity among Indians

61 Consider the following statements regarding Lord Curzon


1. He instituted Universities Commission to go into the entire question of university education in the
country.
2. He passed Ancient Monuments Act, 1904 which made it obligatory on the part of the government
and local authorities to preserve the monuments.

Which of the statements given above is/are correct?


A. 1 only
B. 2 only
C. Both 1 and 2
D. Neither 1 nor 2

Your Answer : B
Correct Answer : C

Answer Justification :

www.insightsactivelearn.com 34
Online Prelims TEST - 3 (SUBJECT WISE)
( InsightsIAS Mock Test Series for UPSC Preliminary Exam 2020 )

He instituted in 1902, a Universities Commission to go into the entire question of university


education in the country. On the basis of the findings and recommendations of the Commission,
Curzon brought in the Indian Universities Act of 1904, which brought all the universities in India
under the control of the government.

He passed a law called the Ancient Monuments Act, 1904 which made it obligatory on the part
of the government and local authorities to preserve the monuments of archaeological importance
and their destruction an offence

62 Consider the following statements regarding Raja Ram Mohan Roy


1. He is considered as the first modern man of India.
2. He aimed at purifying Hinduism and didn’t believe in god.
3. He combined the teachings of the Upanishads, the Bible and the Koran in developing unity among
the people of different religions.

Which of the statements given above is/are correct?


A. 1 only
B. 2 and 3 only
C. 3 only
D. 1 and 3 only

Your Answer : D
Correct Answer : D

Answer Justification :

Statement 1 is correct: Raja Rammohan Roy established the Brahmo Samaj at Calcutta in 1828
in order to purify Hinduism. He is considered as the first ‘modern man of India’

Statement 2 is INCORRECT: He preached monotheism (Belief in one God)

Statement 3 is correct: He combined the teachings of the Upanishads, the Bible and the Koran in
developing unity among the people of different religions.

The work of the Atmiya Sabha was carried on by Maharishi Debendranath Tagore (father of
Rabindranath Tagore), who renamed it as Brahmo Samaj. He turned the Brahmo Samaj into a
leading social organisation of India

63 Consider the following statements.


1. Jagannath Shankar Seth and Bhau Daji were among the active promoters of girls’ schools in
Calcutta.
2. Vishnu Shastri Pandit founded the Widow Remarriage Association
3. Ramabai Ranade founded the Ladies Social Conference.

Which of the statements given above is/are correct?


A. 1, 2 and 3
B. 2 only

www.insightsactivelearn.com 35
Online Prelims TEST - 3 (SUBJECT WISE)
( InsightsIAS Mock Test Series for UPSC Preliminary Exam 2020 )

C. 3 only
D. 2 and 3 only

Your Answer : C
Correct Answer : D

Answer Justification :

Statement 1 is INCORRECT: Jagannath Shankar Seth and Bhau Daji were among the active
promoters of girls’ schools in Maharashtra.

Statement 2 is correct: Vishnu Shastri Pandit founded the Widow Remarriage Association in the
1850s

Statement 3 is correct: Ramabai Ranade founded the Ladies Social Conference (Bharat Mahila
Parishad), under the parent organisation National Social Conference, in 1904 in Bombay.

64 Consider the following statements


1. Henry Vivian Derozio founded Hindu College
2. Raja Ram Mohan Roy wrote the book Satyartha Prakash which contains his ideas.
3. Arya Samaj actively involved in Indian National Movement

Which of the statements given above is/are correct?


A. 1 and 2 only
B. 2 and 3 only
C. 3 only
D. 1 and 3 only

Your Answer : A
Correct Answer : C

Answer Justification :

Statement 1 is INCORRECT: In 1817, Raja Ram Mohan Roy founded the Hindu College (now
Presidency College, Calcutta) along with David Hare, a missionary.

Statement 2 is INCORRECT: Swami Dayanand Saraswathi started the Suddhi movement to bring
back those Hindus who had converted to other religions to its fold. He wrote the book Satyartha
Prakash which contains his ideas

Statement 3 is correct: The Arya Samaj had also spread nationalism. Hundreds of Arya Samaj
patriots, including Lala Lajpat Rai, took part in the Indian freedom struggle.

65 Consider the following statements


1. The Prarthana Samaj was founded in 1867 in Bombay by Dr. Atmaram Pandurang.
2. The Theosophical Society was founded in 1882 in Adyar by Annie Besant.

www.insightsactivelearn.com 36
Online Prelims TEST - 3 (SUBJECT WISE)
( InsightsIAS Mock Test Series for UPSC Preliminary Exam 2020 )

Which of the statements given above is/are correct?


A. 1 only
B. 2 only
C. Both 1 and 2
D. Neither nor 2

Your Answer : C
Correct Answer : A

Answer Justification :

Statement 1 is correct: The Prarthana Samaj was founded in 1867 in Bombay by Dr. Atmaram
Pandurang. It was an off-shoot of Brahmo Samaj. It was a reform movement within Hinduism and
concentrated on social reforms like inter-dining, inter-marriage, widow remarriage and uplift of
women and depressed classes.

Statement 2 is INCORRECT: The Theosophical Society was founded in New York (USA) in 1875
by Madam H.P. Blavatsky, a Russian lady, and Henry Steel Olcott, an American colonel.
They arrived in India and established their headquarters at Adyar in Madras in 1882. Later in 1893,
Mrs. Annie Besant arrived in India and took over the leadership of the Society after the death of
Olcott.

66 Consider the following statements regarding Pandit Ishwar Chandra Vidyasagar


1. He helped J.D. Bethune to establish the Bethune School.
2. He founded the Metropolitan Institution in Bombay.
3. He firmly believed that reform in Indian society could only come about through education.

Which of the statements given above is/are correct?


A. 1 only
B. 1 and 3 only
C. 3 only
D. 1, 2 and 3

Your Answer : D
Correct Answer : B

Answer Justification :

Statement 1 is correct: Vidyasagar founded many schools for girls. He helped J.D. Bethune to
establish the Bethune School.

Statement 2 is INCORRECT: He founded the Metropolitan Institution in Calcutta.

Statement 3 is correct: He rose to be the Head Pandit of the Bengali Department of Fort William
College. He firmly believed that reform in Indian society could only come about through education.

67 Consider the following statements

www.insightsactivelearn.com 37
Online Prelims TEST - 3 (SUBJECT WISE)
( InsightsIAS Mock Test Series for UPSC Preliminary Exam 2020 )

1. The Aligarh Movement was started by Sir Syed Ahmad Khan for the social and educational
advancement of the Muslims in India.
2. He founded a modern school at Aligarh to promote English education among the Muslims.

Which of the statements given above is/are correct?


A. 1 only
B. 2 only
C. Both 1 and 2
D. Neither 1 nor 2

Your Answer : A
Correct Answer : C

Answer Justification :

Both the statements are correct.

The Aligarh Movement was started by Sir Syed Ahmad Khan (1817-98) for the social and
educational advancement of the Muslims in India. He fought against the medieval backwardness
and advocated a rational approach towards religion.

In 1875, he founded a modern school at Aligarh to promote English education among the
Muslims. This had later grown into the Mohammadan Anglo Oriental College and then into the
Aligarh Muslim University.

68 Consider the following statements


1. Periyar opposed Varnashrama
2. The aims of the Self -Respect movement were to uplift the Dravidians and to expose the Brahminical
tyranny.
3. Periyar himself conducted many marriages without any rituals.

Which of the statements given above is/are correct?


A. 1 and 2 only
B. 2 only
C. 1, 2 and 3
D. 2 and 3 only

Your Answer : C
Correct Answer : C

Answer Justification :

All the statements are correct.

Periyar E.V. Ramaswamy was a great social reformer.

E.V.R. opposed the Varnashrama policy followed in the V.V.S. Iyer’s Seranmadevi Gurugulam.
During 1920- 1925 being in the Congrees Party he stressed that Congress should accept communal

www.insightsactivelearn.com 38
Online Prelims TEST - 3 (SUBJECT WISE)
( InsightsIAS Mock Test Series for UPSC Preliminary Exam 2020 )

representation.

Subsequently in 1925, he started the “Self-Respect Movement”. The aims of the ‘Self -Respect
Movement’ were to uplift the Dravidians and to expose the Brahminical tyrany and deceptive
methods by which they controlled all spheres of Hindu life

He encouraged inter-caste marriages. He himself conducted many marriages without any rituals.
Such a marriage was known as “SelfRespect Marriage.” He gave secular names to new born babies.

69 Amongst the following, the earliest to form was

A. The British Indian Association


B. The Bombay Association
C. The Madras Mahajana Sabha
D. Poona Sarvojanik Sabha

Your Answer : B
Correct Answer : A

Answer Justification :

These were some of the early Political Associations that paved the way for organized struggle in
India:

The British Indian Association – 1851 Bengal

The Bombay Association — 1852 Dadabhai Naoroji

East India Association 1856 London

Madras Native Association 1852

Poona Sarvojanik Sabha—1870

The Madras Mahajana Sabha—1884

70 The aim of the Indian Association found by Suredranath Banerjee was to

A. Represent the views of the educated middle class and take united community action
B. Give shape to and establish the Indian National Congress (INC)
C. Advocate constitutional reforms through legislative action
D. Organize seditious movements to dethrone the British Empire

Your Answer : C
Correct Answer : A

Answer Justification :

www.insightsactivelearn.com 39
Online Prelims TEST - 3 (SUBJECT WISE)
( InsightsIAS Mock Test Series for UPSC Preliminary Exam 2020 )

The foundations of the Indian National Movement were laid by Suredranath Banerjee with the
formation of Indian Association at Calcutta in 1876.

The aim of the Association was to represent the views of the educated middle class, inspire
the Indian community to take the value of united action.

71 The Ilbert Bill Controversy is said to be a high watermark in the history of Indian National
Movement. This is because it invoked issues of
1. Racial discrimination between Indian and Europeans
2. Security lapse on the Indian borders, especially the North-Western frontier
3. Suppression of Indian media houses and their nationalization by the Government

Which of the statements given above is/are correct?


A. 1 only
B. 1 and 2 only
C. 2 and 3 only
D. 1 and 3 only

Your Answer : C
Correct Answer : A

Answer Justification :

According to the system of law, a European could be tried only by a European Judge or a European
Magistrate. The disqualification was unjust and it was sought to cast a needless discredit and
dishonour upon the Indian-born members of the judiciary.

C.P. Ilbert, Law Member, introduced a bill in 1883 to abolish this discrimination in
judiciary. But Europeans opposed this Bill strongly. Hence Statement 1 is correct and other
statements are INCORRECT.

72 The Vernacular Press Act in British India was concerned with

A. Promoting local language newspapers for generating local employment


B. Providing for licensing and registration of newspapers published by indigenous tribes
C. Banning all vernacular language publications in India
D. None of the above

Your Answer : C
Correct Answer : D

Answer Justification :

The Vernacular Press Act was passed in 1878 under the Governor Generalship and Viceroyalty of
Lord Lytton, for better control of Indian language newspapers. The purpose of the Act was to
control the printing and circulation of seditious material, specifically that which could produce
disaffection against the British Government in India in the minds of the masses.

www.insightsactivelearn.com 40
Online Prelims TEST - 3 (SUBJECT WISE)
( InsightsIAS Mock Test Series for UPSC Preliminary Exam 2020 )

It was passed on the model of irish press laws. It provided the government extensive rights to
censor report and editorial in vernacular press. At the time the Vernacular Press Act was passed,
there were thirty five vernacular papers in Bengal, including the Amrita Bazar Patrika.

73 Consider the following statements regarding Indian Councils Act, 1861


#88663
1. It initiated the process of decentralisation by restoring the legislative powers to the Bombay and
Madras Presidencies.
2. It empowered the Viceroy to issue ordinances, without the concurrence of the legislative council,
during an emergency.
3. It increased the functions of legislative councils and gave them the power of discussing the budget.

Which of the statements given above is/are correct?


A. 1 and 3 only
B. 2 and 3 only
C. 1 and 2 only
D. 1, 2 and 3

Your Answer : A
Correct Answer : C

Answer Justification :

The Indian Councils Act of 1861 is an important landmark in the constitutional and political history
of India.

Features of the Act of 1861

Statement 1 is correct: It initiated the process of decentralisation by restoring the legislative


powers to the Bombay and Madras Presidencies. It thus reversed the centralising tendency that
started from the Regulating Act of 1773 and reached its climax under the Charter Act of 1833.

Statement 2 is correct: It empowered the Viceroy to issue ordinances, without the concurrence of
the legislative council, during an emergency. The life of such an ordinance was six months.

Statement 3 is INCORRECT: Power of discussing budget was not provided through The Indian
Councils Act of 1861.

74 With reference to the East India Association (London), consider the following statements.
1. It was founded by Surendranath Banerjee.
2. It superseded the London Indian Society.
3. It was an extremist organization that had its main goal of organizing an armed struggle against the
British in India.

Which of the statements given above is/are correct?

www.insightsactivelearn.com 41
Online Prelims TEST - 3 (SUBJECT WISE)
( InsightsIAS Mock Test Series for UPSC Preliminary Exam 2020 )

A. 1 only
B. 2 and 3 only
C. 2 only
D. 1, 2 and 3

Your Answer : A
Correct Answer : C

Answer Justification :

Statement 1 is INCORRECT: It was founded by Dadabhai Naoroji in 1866, in collaboration with


Indians and retired British officials in London.

Statement 2 is correct: It superseded the London Indian Society and was a platform for
discussing matters and ideas about India, and to provide representation for Indians to the
Government.

Statement 3 is INCORRECT: It was not an extremist organization.

75 Which of the following is/are not associated with the cause of women social reform movements in
India?
1. All India Muslim Ladies Conference
2. Arya Samaj in Punjab
3. Pandita Ramabai

Select the correct answer using the codes given below:


A. 1 and 2 only
B. 2 only
C. 1 and 3 only
D. All were associated

Your Answer : B
Correct Answer : D

Answer Justification :

All the above organisations/Personalities were associated.

The All-India Muslim Ladies Conference (Anjuman-E-Khawatn-E-Islam) was founded in 1914. Indian
reformers debated not just in public meetings but through public media like newspapers and
journals. Translations of writings of social reformers from one Indian language to another took
place.

Within India, social reformers from Punjab and Bengal exchanged ideas with reformers from
Madras and Maharashtra. Keshav Chandra Sen of Bengal visited Madras in 1864.

Pandita Ramabai travelled to different corners of the country. Some of them went to other
countries. Modern social organisations like the Brahmo Samaj in Bengal and Arya Samaj in Punjab

www.insightsactivelearn.com 42
Online Prelims TEST - 3 (SUBJECT WISE)
( InsightsIAS Mock Test Series for UPSC Preliminary Exam 2020 )

were set up.

76 Consider the following statements regarding Punch Mission


1. It is scheduled to be launched by NASA in co-operation with ISRO
2. It consists constellation of four suitcase-sized microsats.
3. It focused on understanding the transition of particles from the Sun’s outer corona to the solar
wind.

Which of the statements given above is/are correct?


A. 1 and 2 only
B. 2 only
C. 3 only
D. 2 and 3 only

Your Answer : D
Correct Answer : D

Answer Justification :

Statement 1 is INCORRRECT: NASA has selected Texas-based Southwest Research Institute to


lead its PUNCH mission which will image the Sun. This is a landmark mission that will image
regions beyond the Sun’s outer corona. Dipankar Banerjee, solar physicist from Indian Institute of
Astrophysics is also a Co-Investigator of the PUNCH mission. ISRO is not involved in the project.

Statement 2 and 3 are correct:

PUNCH, which stands for “Polarimeter to Unify the Corona and Heliosphere,” is focused on
understanding the transition of particles from the Sun’s outer corona to the solar wind that fills
interplanetary space.

PUNCH will consist of a ‘constellation’ of four suitcase-sized microsats that will orbit the Earth in
formation and study how the corona, which is the atmosphere of the Sun, connects with the
interplanetary medium. The mission is expected to be launched in 2022.

The mission will image and track the solar wind and also the coronal mass ejections – which are
huge masses of plasma that get thrown out of the Sun’s atmosphere. The coronal mass ejections can
affect and drive space weather events near the Earth.

https://www.thehindu.com/sci-tech/science/indian-scientist-to-be-co-i-for-nasas-punch-mission/articl
e28228527.ece

77 Mahatma Gandhi Information Technology & Biotech Park, recently seen in news, is inaugurated in

A. Cote d’Ivoire
B. Sierra Leone
C. Burkina Faso
D. South Africa

www.insightsactivelearn.com 43
Online Prelims TEST - 3 (SUBJECT WISE)
( InsightsIAS Mock Test Series for UPSC Preliminary Exam 2020 )

Your Answer : D
Correct Answer : A

Answer Justification :

Mahatma Gandhi IT and Biotechnology Park (MGIT-BP) is a dedicated free trade zone built with
India’s assistance, has been inaugurated in Cote d’Ivoire. The MGIT-BP is being built with India’s
assistance through EXIM Bank Lines of Credit of US$ 20 million.

https://www.mea.gov.in/press-releases.htm?dtl/31518/Mahatma+Gandhi+Information+Technology
+amp+Biotech+Park+Inaugurated+in+Cte+dIvoire

78 Consider the following statements regarding One Nation One Ration Card Scheme
1. Migrant workers will be able to buy subsidised rice and wheat from any ration shop in the country
after the implementation of the scheme.
2. Presently, no state in India offers portability scheme for ration card.

Which of the statements given above is/are correct?


A. 1 only
B. 2 only
C. Both 1 and 2
D. Neither 1 nor 2

Your Answer : C
Correct Answer : A

Answer Justification :

Statement 1 is correct: ‘One Nation One Ration Card’ scheme, which will allow portability of food
security benefits, will be available across the country from July 1, 2020. This means poor migrant
workers will be able to buy subsidised rice and wheat from any ration shop in the country, so long
as their ration cards are linked to Aadhaar.

Statement 2 is INCORRECT: Presently, Ten States -- Andhra Pradesh, Gujarat, Haryana,


Jharkhand, Karnataka, Kerala, Maharashtra, Rajasthan, Telangana and Tripura – already offer this
portability.

https://www.thehindu.com/news/national/centre-gives-1-year-deadline-to-states-for-rolling-out-one-n
ation-one-ration-card/article28227599.ece

79 Consider the following statements


1. The Kartarpur Corridor is a proposed border corridor connecting the Sikh shrines of Dera Baba
Nanak Sahib and Gurdwara Darbar Sahib Kartarpur.
2. Corridor is intended to allow religious devotees from India to visit the Gurdwara in Kartarpur
without a visa.
3. The Kartarpur Corridor was first proposed in early 1999 as part of the Delhi–Lahore Bus diplomacy.

www.insightsactivelearn.com 44
Online Prelims TEST - 3 (SUBJECT WISE)
( InsightsIAS Mock Test Series for UPSC Preliminary Exam 2020 )

Which of the statements given above is/are correct?


A. 3 only
B. 2 only
C. 1 and 2 only
D. 1, 2 and 3

Your Answer : D
Correct Answer : D

Answer Justification :

Statement 1 is correct: The Kartarpur Corridor is a proposed border corridor between the
neighbouring nations of India and Pakistan, connecting the Sikh shrines of Dera Baba Nanak
Sahib (located in Punjab, India) and Gurdwara Darbar Sahib Kartarpur (in Punjab, Pakistan).

Statement 2 is correct: Currently under planning, the corridor is intended to allow religious
devotees from India to visit the Gurdwara in Kartarpur, 4.7 kilometres (2.9 miles) from the Pakistan-
India border, without a visa.

Statement 3 is correct: The Kartarpur Corridor was first proposed in early 1999 by the prime
ministers of Pakistan and India, Nawaz Sharif and Atal Bihari Vajpayee, respectively, as part of
the Delhi–Lahore Bus diplomacy.

https://www.thehindu.com/news/national/kartarpur-corridor-officials-from-pak-india-to-discuss-moda
lities/article28427180.ece

80 Consider the following statements regarding Dragonfly Mission


1. Dragonfly aims to search for signs of microbial alien life on Mars
2. Dragonfly mission is a part of NASA’s New Frontiers program

Which of the statements given above is/are correct?


A. 1 only
B. 2 only
C. Both 1 and 2
D. Neither 1 nor 2

Your Answer : B
Correct Answer : B

Answer Justification :

Statement 1 is INCORRECT: Dragonfly aims to search for signs of microbial alien life on
Saturn’s moon Titan, while navigating its earth-like gravity and aerodynamics in the process.

The mission will succeed NASA’s Cassini probe, which ended its 13-year mission orbiting Saturn in
September 2017 by diving into Saturn’s atmosphere.

Statement 2 is correct: Dragonfly mission is a part of NASA’s New Frontiers program, which

www.insightsactivelearn.com 45
Online Prelims TEST - 3 (SUBJECT WISE)
( InsightsIAS Mock Test Series for UPSC Preliminary Exam 2020 )

includes a series of space exploration missions, which are being conducted with the purpose of
researching several of the Solar System bodies, including the dwarf planet Pluto.

https://www.insightsonindia.com/2019/07/10/nasa-to-launch-dragonfly/

81 Consider the following statements regarding Nirbhaya Fund


1. The fund is created by the Ministry of Finance, dedicated for implementation of initiatives aimed at
enhancing the safety and security for women in the country.
2. It is a non-lapsable corpus fund.
3. It is being administered by the Ministry of Women and Child Development

Which of the statements given above is/are correct?


A. 2 only
B. 1 and 2 only
C. 1, 2 and 3
D. 1 and 3 only

Your Answer : C
Correct Answer : B

Answer Justification :

Statement 1 is Correct: Nirbhaya Fund was created by the Ministry of Finance in 2013, dedicated
for implementation of initiatives aimed at enhancing the safety and security for women in the
country.

Statement 2 is Correct: It is a non-lapsable corpus fund.

Statement 3 is INCORRECT: It is being administered by the Department of Economic Affairs,


Ministry of Finance. Ministry of Women and Child Development is the nodal authority for
appraising/recommending the proposals/schemes to be funded under fund.

Some of the schemes under the funds:

One Stop Centre Scheme, also known as “Sakhi Centres” aims at establishing centres to facilitate
women affected by violence. Mahila Police Volunteers (MPVs) will act as a link between police
and community and help women in distress.

https://www.thehindu.com/news/national/only-20-of-nirbhaya-fund-has-been-used-by-states-until-201
8/article28230097.ece

82 Consider the following statements regarding Global Financial Stability Report


1. It is a semiannual report that assesses the stability of global financial markets and emerging market
financing.
2. It is published by International Monetary Fund.

Which of the statements given above is/are correct?

www.insightsactivelearn.com 46
Online Prelims TEST - 3 (SUBJECT WISE)
( InsightsIAS Mock Test Series for UPSC Preliminary Exam 2020 )

A. 1 only
B. 2 only
C. Both 1 and 2
D. Neither 1 nor 2

Your Answer : C
Correct Answer : C

Answer Justification :

Both the statements are correct

What is the Global Financial Stability Report?

The Global Financial Stability Report (GFSR) is a semiannual report by the International
Monetary Fund (IMF) that assesses the stability of global financial markets and emerging-market
financing. It is released twice per year, in April and October.

The GFSR focuses on current conditions, especially financial and structural imbalances, which could
risk an upset in global financial stability and access to financing by emerging-market countries. It
emphasizes the ramifications of financial and economic imbalances that are highlighted in one of
the IMF's other publications, the World Economic Outlook.

https://www.investopedia.com/terms/g/gfsr.asp

83 Consider the following statements regarding Particularly Vulnerable Tribal Groups (PVTGs)
1. PVTGs reside in 18 States and 2 union territories
2. Highest number of PVTGs are found in Madhya Pradesh
3. The Ministry of Tribal Affairs implements the Scheme of “Development of Particularly Vulnerable
Tribal Groups (PVTGs)” exclusively for them.

Which of the statements given above is/are correct?


A. 1 and 3 only
B. 3 only
C. 2 and 3 only
D. 1 and 2 only

Your Answer : A
Correct Answer : B

Answer Justification :

Statement 1 is INCORRECT: PVTGs reside in 18 States and UT of A&N Islands.

Statement 2 is INCORRECT: Among the 75 listed PVTG’s the highest number are found in
Odisha (13), followed by Andhra Pradesh (12), Bihar including Jharkhand (9) Madhya Pradesh
including Chhattisgarh (7) Tamil Nadu (6) Kerala and Gujarat having five groups each. The
remaining PVTGs live in West Bengal (3) Maharashtra (3), two each in Karnataka and Uttarakhand

www.insightsactivelearn.com 47
Online Prelims TEST - 3 (SUBJECT WISE)
( InsightsIAS Mock Test Series for UPSC Preliminary Exam 2020 )

and one each in Rajasthan, Tripura and Manipur.

All the four tribal groups in Andamans, and one in Nicobar Islands, are recognised as PVTGs.

Statement 3 is Correct: The Ministry of Tribal Affairs implements the Scheme of “Development of
Particularly Vulnerable Tribal Groups (PVTGs)” exclusively for them.

Under the scheme, Conservation-cum-Development (CCD)/Annual Plans are to be prepared by each


State/UT for their PVTGs based on their need assessment, which are then appraised and approved
by the Project Appraisal Committee of the Ministry. Activities for development of PVTGs are
undertaken in Sectors of Education, Health, Livelihood and Skill Development, Agricultural
Development, Housing & Habitat, Conservation of Culture etc.

https://tribal.nic.in/pvtg.aspx

http://pib.nic.in/newsite/PrintRelease.aspx?relid=178257

84 Consider the following statements regarding “The Ocean Cleanup”


1. It develops advanced technologies to rid world’s oceans of plastic.
2. It utilizes the ocean currents to clean up half the Great Pacific Garbage Patch in 5 years’ time.

Which of the statements given above is/are correct?


A. 1 only
B. 2 only
C. Both 1 and 2
D. Neither 1 nor 2

Your Answer : C
Correct Answer : C

Answer Justification :

Both the statements are correct.

The Ocean Cleanup is a non-profit organization, developing advanced technologies to rid the
world’s oceans of plastic.

By utilizing the ocean currents to our advantage, our passive drifting systems are estimated to clean
up half the Great Pacific Garbage Patch in 5 years’ time.

About Ocean Cleanup:

The Ocean Cleanup is designing and developing the first feasible method to rid the world’s oceans
of plastic. Every year, millions of tons of plastic enter the ocean. A significant percentage of this
plastic drifts into large systems of circulating ocean currents, also known as gyres. Once trapped in
a gyre, the plastic will break down into microplastics and become increasingly easier to mistake for
food by sea life.

Going after it with vessels and nets would be costly, time-consuming, labor-intensive and lead to

www.insightsactivelearn.com 48
Online Prelims TEST - 3 (SUBJECT WISE)
( InsightsIAS Mock Test Series for UPSC Preliminary Exam 2020 )

vast amounts of carbon emission and by-catch. That is why The Ocean Cleanup is developing a
passive system, moving with the currents – just like the plastic – to catch it.

https://theoceancleanup.com/about/

85 Consider the following pairs


List I List II
1.
Bhashan Char Bangladesh
2.
Yazidis Yemen
3.
Tashkent Kazakhstan

Which of the pairs given above is/are matched correctly?


A. 1 only
B. 2 and 3 only
C. 2 only
D. None

Your Answer : C
Correct Answer : A

Answer Justification :

Rohingya refugees living in Bangladesh are seemingly in the dark on the benefits, risks, and
probable side-effects of relocation to the remote island of Bhashan Char, located in an estuary of
the Meghna River.

The Bangladesh government assures a comfortable living for the relocated Rohingyas on the island,
though the community appears to be unconvinced.

Yazidis are a mostly Kurmanji-speaking ethnoreligious group, or an ethnic Kurdish minority


indigenous to Iraq, Syria and Turkey who are strictly endogamous.

Tashkent is the capital city of Uzbekistan. It’s known for its many museums and its mix of
modern and Soviet-era architecture.

86 Consider the following pairs


List I List II
1. Baitarani River - Odisha
2. Salma Dam - Pakistan
3. Kuki Tribes - Mizoram

Which of the pairs given above is/are matched correctly?


A. 1, 2 and 3
B. 1 only
C. 3 only

www.insightsactivelearn.com 49
Online Prelims TEST - 3 (SUBJECT WISE)
( InsightsIAS Mock Test Series for UPSC Preliminary Exam 2020 )

D. 1 and 3 only

Your Answer : A
Correct Answer : D

Answer Justification :

The Baitarani River or River Baitarani is one of six major rivers of Odisha, India. Venerated
in popular epics and legends, the Baitarani River is a source of water for agricultural irrigation.

Salma Dam is located in Afghanistan.

The Kukis, also known as the Chin.They are one of the hill tribe Zo people. In the Chin State of
Myanmar and as Mizo in the Indian state of Mizoram are a number of related Tibeto-Burman
tribal peoples spread throughout the northeastern states of India.

87 Consider the following statements regarding National Anti-Profiteering Authority (NAA)


1. National Anti-profiteering Authority is the institutional mechanism under GST law to check the
unfair profit-making activities by the trading community.
2. Core function of NAA is to ensure that the benefits of the reduction in GST rates on goods and
services made by GST Council ultimately passed to consumers.

Which of the statements given above is/are correct?


A. 1 only
B. 2 only
C. Both 1 and 2
D. Neither 1 nor 2

Your Answer : C
Correct Answer : C

Answer Justification :

Both the statements are correct.

The National Anti-Profiteering Authority (NAA) has been constituted under Section 171 of the
Central Goods and Services Tax Act, 2017 to ensure that the reduction in rate of tax or the benefit
of input tax credit is passed on to the recipient by way of commensurate reduction in prices.
Further, the following steps have been taken by the NAA to ensure that customers get the full
benefit of tax cuts:

1. Holding regular meetings with the Zonal Screening Committees and the Chief Commissioners
of Central Tax to stress upon consumer awareness programmes;

2. Launching a helpline to resolve the queries of citizens regarding registration of complaints


against profiteering.

www.insightsactivelearn.com 50
Online Prelims TEST - 3 (SUBJECT WISE)
( InsightsIAS Mock Test Series for UPSC Preliminary Exam 2020 )

3. Receiving complaints through email and NAA portal.

4. Working with consumer welfare organizations in order to facilitate outreach activities.

http://www.pib.nic.in/Pressreleaseshare.aspx?PRID=1559128

88 Glyphosate, often seen in the news, is related to

A. Multidrug Resistance Tuberculosis


B. Weedicide
C. Bio-chemical agent to remove oil spills
D. None of the above

Your Answer :
Correct Answer : B

Answer Justification :

https://www.business-standard.com/article/current-affairs/gujarat-firms-sell-monsanto-s-bt3-seeds-c
arcinogenic-herbicide-despite-ban-118031300141_1.html

89 Which of the following National Parks is/are part of Western Ghats?


1. Bandipur National Park
2. Gir National Park
3. Eravikulam National Park
4. Tadoba Andhari National Park

Select the correct answer using the code given below


A. 1, 3 and 4
B. 2, 3 and 4
C. 1 and 3 only
D. 1 only

Your Answer : A
Correct Answer : C

Answer Justification :

Bandipur National Park established in 1974 as a tiger reserve under Project Tiger, is a national
park located in the Indian state of Karnataka, which is the state with the highest tiger population in
India.

Gir National Park and Wildlife Sanctuary, also known as Sasan Gir, is a forest and wildlife
sanctuary near Talala Gir in Gujarat, India. It is part of the Kathiawar-Gir dry deciduous forests
ecoregion

www.insightsactivelearn.com 51
Online Prelims TEST - 3 (SUBJECT WISE)
( InsightsIAS Mock Test Series for UPSC Preliminary Exam 2020 )

Eravikulam National Park is a 97 km2 national park located along the Western Ghats in the
Idukki district of Kerala in India.

Tadoba Andhari Tiger Reserve is located in Chandrapur district of Maharashtra state in India. It
is Maharashtra's oldest and largest national park. It is not a part of Western Ghats.

90 Consider the following statements


1. Accessible India Campaign aims to provide accessibility to banking services in remote areas.
2. Government e-Marketplace (GeM) facilitates online procurement of common use Goods & Services
required by various Government Departments / Organizations / PSUs.

Which of the statements given above is/are correct?


A. 1 only
B. 2 only
C. Both 1 and 2
D. Neither 1 nor 2

Your Answer : C
Correct Answer : B

Answer Justification :

Statement 1 is INCORRECT: Accessible India Campaign or Sugamya Bharat Abhiyan is a


program which is set to be launched to serve the differently-able community of the country. The
program comes with an index to measure the design of disabled-friendly buildings and human
resource policies.

Statement 2 is correct: Government e-Marketplace (GeM) facilitates online procurement of


common use Goods & Services required by various Government Departments / Organizations /
PSUs.

http://vikaspedia.in/e-governance/online-citizen-services/government-to-business-services-g2b/gover
nment-e-marketplace

91 Consider the following statements regarding Block Chain Technology


1. It is a time-stamped series of immutable record of data that is managed by cluster of computers not
owned by any single entity.
2. The blockchain network has no central authority.
3. It carries no transaction cost.

Which of the statements given above is/are correct?


A. 1 and 2 only
B. 1 and 2 only
C. 3 only
D. 1, 2 and 3

Your Answer :

www.insightsactivelearn.com 52
Online Prelims TEST - 3 (SUBJECT WISE)
( InsightsIAS Mock Test Series for UPSC Preliminary Exam 2020 )

Correct Answer : D

Answer Justification :

All the statements are correct.

A blockchain is, in the simplest of terms, a time-stamped series of immutable record of


data that is managed by cluster of computers not owned by any single entity. Each of these
blocks of data (i.e. block) are secured and bound to each other using cryptographic principles (i.e.
chain).

The blockchain network has no central authority — it is the very definition of a democratized
system. Since it is a shared and immutable ledger, the information in it is open for anyone and
everyone to see.

A blockchain carries no transaction cost. (An infrastructure cost yes, but no transaction cost.)

https://blockgeeks.com/guides/what-is-blockchain-technology/

92 World Population Prospects, often seen in news, is published by

A. United Nations
B. World Economic Forum
C. World Bank
D. The Organisation for Economic Co-operation and Development

Your Answer : C
Correct Answer : A

Answer Justification :

The World Population Prospects 2019 is published by the Population Division of the UN
Department of Economic and Social Affairs, provides a comprehensive overview of global
demographic patterns and prospects. The study concluded that the world’s population could reach
its peak around the end of the current century, at a level of nearly 11 billion.

The world’s population is expected to increase by 2 billion persons in the next 30 years, from 7.7
billion currently to 9.7 billion in 2050

https://www.un.org/development/desa/publications/world-population-prospects-2019-highlights.html

93 The Kimberly Process, often seen in news, is related with

A. Prevention of trafficking of wild animals.


B. Mediation process between the North Korea and the South Korea
C. Prevention of conflict diamonds entering into market
D. Promotion of nuclear-non proliferation

www.insightsactivelearn.com 53
Online Prelims TEST - 3 (SUBJECT WISE)
( InsightsIAS Mock Test Series for UPSC Preliminary Exam 2020 )

Your Answer : D
Correct Answer : C

Answer Justification :

The Kimberley Process Certification Scheme (KPCS) is the process established in 2003 to prevent
"conflict diamonds" from entering the mainstream rough diamond market by United Nations
General Assembly Resolution.

https://www.thehindu.com/business/india-to-actively-curb-conflict-diamonds/article28022481.ece

94 Consider the following statements regarding UN Security Council


1. It is the only UN body with the authority to issue binding resolutions to member states.
2. The presidency of the Council is held by each of the members in turn for one month, following the
English alphabetical order of the Member States names.
3. It was established in 1945 and headquartered in Geneva.

Which of the statements given above is/are correct?


A. 1 only
B. 2 and 3 only
C. 1, 2 and 3
D. 1 and 2 only

Your Answer : B
Correct Answer : D

Answer Justification :

Statement 1 is correct: All members of the United Nations agree to accept and carry out the
decisions of the Security Council. While other organs of the United Nations make recommendations
to member states, only the Security Council has the power to make decisions that member states
are then obligated to implement under the Charter.

Statement 2 is correct: The presidency of the Council is held by each of the members in turn for
one month, following the English alphabetical order of the Member States names.

Statement 3 is INCORRECT: It was established in 1945 and headquartered in New York.

https://www.un.org/securitycouncil/content/what-security-council

https://www.un.org/securitycouncil/content/presidency

95 Consider the following statements regarding Superconductivity.


1. Superconductivity is a state in which a material shows very high electrical resistance.
2. Superconducting materials can save huge amounts of energy, and be used to make highly efficient
electrical appliances.
3. Superconductivity is possible only at high temperatures.

www.insightsactivelearn.com 54
Total Marks : 200
Online Prelims TEST - 3 (SUBJECT WISE)
( InsightsIAS Mock Test Series for UPSC Preliminary Exam 2020 ) Mark Scored : 34

Which of the statements given above is/are not correct?


A. 1 and 2 only
B. 2 and 3 only
C. 1 and 3 only
D. 1, 2 and 3

Your Answer : B
Correct Answer : C

Answer Justification :

Superconductivity is a phenomenon in which the resistance of the material to the electric current
flow is zero.

Because of zero resistance, superconducting materials can save huge amounts of energy, and be
used to make highly efficient electrical appliances.

Superconductivity is largely possible at low temperature. Metallic superconductors usually


have transition temperatures (temperatures below which they are superconductive) below 30 K
(−243.2 °C) and must be cooled using liquid helium in order to achieve superconductivity.

https://www.britannica.com/science/superconductivity

96 Consider the following statements


1. Article 341 of the Constitution provides certain privileges and concessions to the members of
Scheduled Castes.
2. President alone is vested with the power to include or exclude any entry in the Scheduled Castes
(SC) list.
3. There is provision for the reservation of Scheduled Castes both in the Lok Sabha and Rajya Sabha.

Which of the statements given above is/are correct?


A. 1 only
B. 2 only
C. 2 and 3 only
D. 1 and 3 only

Your Answer : A
Correct Answer : A

Answer Justification :

Article 341 of the Constitution provides certain privileges and concessions to the members of
Scheduled Castes.

Statement 2 is INCORRECT: Parliament alone is vested with the power to include or


exclude any entry in the SC list under Article 341 of the Constitution.

Statement 3 is INCORRECT: Reservation is there only in the Lok Sabha.

www.insightsactivelearn.com 55
Total Marks : 200
Online Prelims TEST - 3 (SUBJECT WISE)
( InsightsIAS Mock Test Series for UPSC Preliminary Exam 2020 ) Mark Scored : 34

97 Consider the following statements about Investigation Tracking System for Sexual Offences (ITSSO).
1. ITSSO is an online module available to law enforcement agencies at National, State and District
levels.
2. ITSSO leverages the Crime and Criminal Tracking Network & Systems (CCTNS) which is a
nationwide network enabling sharing of information between Police Stations, State officers and
security agencies.

Which of the statements given above is/are correct?


A. 1 only
B. 2 only
C. Both 1 and 2
D. Neither 1 nor 2

Your Answer : C
Correct Answer : C

Answer Justification :

Both the statements are correct.

Investigation Tracking System for Sexual Offences (ITSSO) is meant for Law Enforcement Agencies
in the country. The Investigation Tracking System for Sexual Offences (ITSSO) is an online module
available to law enforcement agencies at all levels- National, State, District and Police Station that
allows State to undertake real-time monitoring and management for completion of investigation in
rape cases in 2 months.

It leverages the existing CCTNS data base, which covers nearly 15000 police stations in the
country. ITSSO would greatly strengthen States ability for analytics and prognosis for timely
investigation and prosecution in rape cases. ITSSO is one of such measures developed by MHA
towards Smart Policing.

98 Consider the following pairs


Species Habitat
1. Bhupathy’s shieldtail Western Ghats
2. Small Woodbrown Butterfly Khanchendzonga National Park.
3. Pondicherry Shark East Godavari River Estuarine

Which of the pairs given above is/are matched correctly?


A. 1 only
B. 2 and 3 only
C. 3 only
D. 1, 2 and 3

Your Answer : C
Correct Answer : D

Answer Justification :

www.insightsactivelearn.com 56
Online Prelims TEST - 3 (SUBJECT WISE)
( InsightsIAS Mock Test Series for UPSC Preliminary Exam 2020 )

All the pairs have matched correctly.

Just 40 cm long and iridescent brown, Bhupathy’s shieldtail is the latest addition to the snake
fauna of the Western Ghats.

The Small Woodbrown Butterfly species was discovered at Bakhim in Khanchendzonga National
Park after 120 years by researchers from Sikkim University.

99 Consider the following statements


1. The Convention concerning Minimum Age for Admission to Employment, is a Convention adopted in
1973 by the International Labour Organization.
2. It requires ratifying states to pursue a national policy designed to ensure the effective abolition of
child labour.
3. India joined International Labour Organization as a member state in 1940’s.

Which of the statements given above is/are correct?


A. 1 only
B. 1 and 2 only
C. 3 only
D. 2 and 3 only

Your Answer : B
Correct Answer : B

Answer Justification :

India is a founder member of the International Labour Organization (ILO), which came into
existence in 1919.

https://www.humanium.org/en/minimum-age-convention/

100 Israel has borders with which of the following countries.


1. Saudi Arabia
2. Egypt
3. Turkey
4. Jordan
5. Lebanon

Select the correct answer using the code given below


A. 1, 2, 3, 4 and 5
B. 2, 3, 4 and 5 only
C. 4 and 5 only
D. 2, 4 and 5 only A initiative to provide
watermarks and
Your Answer : B
bookmarks free pdfs to you.
Correct Answer : D Share and Subscribe our telegram channel
@visionpt3652019
www.insightsactivelearn.com 57
https://t.me/visionpt3652019
Online Prelims TEST - 3 (SUBJECT WISE)
( InsightsIAS Mock Test Series for UPSC Preliminary Exam 2020 )

Answer Justification :

www.insightsactivelearn.com 58

You might also like